SlideShare a Scribd company logo
1 of 33
Comprehensive ป 5 วันที่ 13 กุมภาพันธ 2553
              ณ โรงพยาบาลศิริราช คณะแพทยศาสตรศิริราชพยาบาล
               ขอสอบแบงออกเปน รอบเชา 150 ขอ 8.30 – 12.00 น.
                     และรอบบาย 150 ขอ 13.00 – 16.30 น.

                              ลักลอบออกมาอยางถูกตอง
                                    - - BY SI116 - -
                   ขอบคุณเพื่อนๆ SI116 ทุกคนที่ชวยกันจําออกมานะ
                  ขอบคุณเพื่อนๆ ที่ชวยกันพิมพ รวบรวม และเรียบเรียง
พี่สาระ194 กานต008 กานต011 อรณ015 จอย035 ออบ043 ปง056 ชา058 โอม69 ธัญ079
    นอร082 แพร085 แนท089 ปาน090 แปก097 ปอ108 พลอย118 แบงค144 ถิงถิง147
            นุย157 เจ160 หยก161 มิ้ม173 บอย180 ตุกตา183 โย191 ไปฟ201
           ญา217 แน็ต221 เบลล222 พลอย226 ปลาม230 บิ๋ว242 กาวหนาPI*
        และคนอื่นๆถาลืมไปตองขออภัยดวยนะ ..สุดทาย..ปกนารักๆ โดย กิ๊บ106
                  แถมๆๆ สถิติของการสอบจากผูเขาสอบจํานวน 348 คน
                      Max 74.92%, Min 35.12%, Mean 59.87%,
      Reliability          0.82, Standard error of measurement (SEM) 7.38%
 เกณฑการตัดสินผลการสอบ: เกณฑขั้นต่ํา 60% นําคา SEM มาพิจารณาคะแนนต่ําสุดที่
สามารถสอบผานโดยใชระดับความเชื่อมั่น 99% ดังนั้น คะแนนต่ําสุดที่สอบผาน 53.63 %
- - - - - - - - - - - - - - - - 01 Medicine - - - - - - - - - - - - - - - -
              Contents                   <<<<< Hematology >>>>>
                                         1. ผูปวยหญิงอายุ 25 ป มาพบแพทยดวยอาการประจําเดือนมามาก ครั้งละ 10 วัน มา 2
01   Medicine                    P. 1    เดือน มีเลือดออกตามไรฟนมา 2 สัปดาห ตรวจรางกายพบซีดเล็กนอย ไมเหลือง ตับ-
                                         มาม-ตอมน้ําเหลืองไมโต มีจุดเลือดออกใตผิวหนังตามแขนขา CBC = Hb 10 , Hct 30%
02   Pediatrics                  P. 20   WBC 6000 , N 75% , L20% ,MCV 71 fl, WBC 6000, Plt 10000 รักษา ???
                                                    A. azathriopine               B. dexamethazone
03   Obstetrics and Gynecology   P. 36              C. vincristine                D. platelet transfusion
                                                    E. IVIG
04   Surgery                     P. 40   2. ลูกคนแรกเปนHbH ตรวจพอ Hb13 MCV65 reticulocyte count1% HbA92%
                                         HbA25.5% แม Hb11 MCV60 reticulocyte count0.5% HbA91% HbE19% ลูกคนตอไปจะ
05   Orthopaedics and rehab      P. 43   มีโอกาสไมเปนโรคและพาหะของโรคกี่เปอรเซ็นต
                                                    A. 0%                         B. 12.5%
06   Eye and ENT                 P. 47              C. 25%                        D. 37.5%
                                                    E. 50%
07   Psychiatry                  P. 51   3. ผูปวยชายอายุ 70 ป มีตอมน้ําเหลืองที่ คอโต 2 ขาง มีซีด เหลืองเล็กนอย ตรวจรางกาย
                                         ปกติ ตับปกติ ยกเวนมามโต
08   Forensic Med and Ethics     P. 55              A. lymphoma with bone metastatis
                                                    B. diffused TB
09   Miscellaneous               P. 60              C. CLL
                                                    D. CML with extramedullary blastic formation




                                               Comprehensive - Medicine                                                             หนาที่ 1
4. มารดาคลอดบุตร ตายตั้งแตแรกคลอด ทารกบวม แม Hb 11 MCV 60 พอ Hb 12.5                                   C. LDH                         D. ดู blood smear
       MCV 55 ขอใดนาจะเปน Hb typing ของพอ                                                          9. หญิง มีซีด เหลือง plt. ต่ํา จะทําอะไร
                   A. - -/α α                                B. - α/α α                                          c.direct coomb test
                   C. - -/ - α                               D. - -/- α cs
                   E. - -/- -                                                                          <<<<< GI >>>>>
       5. ผูปวยมาดวย RUQ pain & moderate jaundice มา 2 วัน ตรวจ U/S พบ common bile                  10. ผูปวยหญิง ถายอุจจาระเละ 3 ครั้ง/วัน มา 2 ป มักเปนเวลากินเผ็ด นม ครีม ปวดทอง มี
       duct stone ไดใหการรักษาดวย ceftriaxone มา 5 วัน อาการไมดีขึ้น                               ลมมากในทอง ผายลม
       ครั้งนี้ตรวจพบ PT prolong , normal aPTT ควรใหการรักษาเบื้องตนอยางไร                                     A. Lactose intolerance        B. IBS              C. CA colon
                   A. vitamin K                              B. FFP                                    11. ผูปวยหญิงอายุ 30 ป ปวดทองนอยขางขวามา 4 ชั่วโมง ปวดมากทันที ไมราวไปที่ใด
                   C. observe                                                                          N/V 1-2 ครั้ง มีถายเหลว 1 ครั้ง ไมมีปสสาวะแสบขัด ไมมีตกขาวผิดปกติ ไมมีไข LMP 1
       6. ชายแก มาดวยออนเพลีย ไข coomb +                                                           เดือนกอน ตรวจรางกาย T 37oC , not pale , no jaundice , abdomen : flat, mild tenderness
                   A. G-6 PD                                                                           at RLQ , CVA not tenderness , CBC : Hct. 37% , WBC 9,000 cell/mm3 , Plt. 250,000
                   B. CLL with AIHA                                                                    /mm3 UA : Sp.Gr. 1.020 , WBC 2-3 /HP , RBC 5-10 /HP จงวินิจฉัยโรค
                   C. Acute leukemia                                                                              A. acute gastroenteritis      B. acute appendicitis
       7. ผูหญิงกินมังสวิรัติ เครงมาก (ไมกินหมู,ไข) กินเตาหู ถั่ว ธัญพืช กินผักใบเขียว ผูปวย              C. acute pyeronephritis       D. ureteric stone
       เสี่ยงที่จะเกิดภาวะใด                                                                                      E. ectopic pregnancy
                   A. folate def.                B. B12 def.                                           12. ช 50 yr ดื่มเหลาขาว ½ ขวด /วัน มา 10 ป ทองบวม ขาโต ตัวเหลือง ตาเหลือง
                   C. Fe def.                    D. ขาด Ca                                             abdomen soft not tender liver not palpable spleen 2 FB shifting dullness
                   E. ขาด protein                                                                      +fluid thrill + LFT TB DB สูง SGOT SGPT ปกติ albumin ปกติ ALP สูง Glo 4+
       8. ผูปวยหญิงอายุ 50 ป มี petechiae ที่ขาทั้ง 2 ขาง + มีเลือดออกตามไรฟน PE: ปกติ ;          Manage ที่ไมเหมาะสมคือ
       CBC : RBC ปกติ , WBC ปกติ(N70%, L15%, E or Mวะ?? 15%), Plt. 15000 ตรวจอะไร                                 A. ERCP                       B. Ultrasound liver
       เพิ่มเติมเพื่อชวยวินิจฉัย                                                                                 C. EGD                        D. Abdominal paracentsis
                   A. bleeding time              B. coagulogram                                        13. แสบรอนแนนหนาอกมา 1 m, ECG: WNL, มักเปนหลังอาหาร กอนนอน Mx

Comprehensive - Medicine                                                              หนาที่ 2               Comprehensive - Medicine                                                            หนาที่ 3
A. simethicone               B. omeprazole                                                 A. urine                    B. ANA
                     C. antacid                   D. oral nitroglycerine                                        C. RF
         14. hepatitis C มีไข , ascites , ขาบวม SAAG มากกวาเทากับ 1.1 จะรักษาอยางไร             18. หญิง30ป ปวดขอนิ้วมือและขอมือ2ขาง 4 เดือน ตรวจพบ symmetrical arthritis of both
                     A. ให albumin                                                                 wrists MCP, PIP joint both hand และพบ joint space narrowing & ulnar styloid erosion
                     B. ceftriaxone 15 d + somatostatin IV                                          จงใหการวินิจฉัย
                     C. cipoflox oral + teripressin IV                                                          A. Osteoarthritis           B. Reumatoid arthritis
                     D. ceftriaxone 29 d                                                                        C. Reactive arthritis       D. SLE
         15. ผูชายอายุ 50 ป มีอาการ cirrhosis pitting edema 2+ , jaundice , spider nevi สงตรวจ               E. .scleroderma
         อะไรเพื่มเติม                                                                              19. หญิงอายุ25ป มีอาการปวดขอนิ้ว ขอมือ ศอก เขาทั้งสองขาง มาประมาณ4เดือน ขอติด
                     A. HBS Ag , Anti HCV , Anti HBC                                                ตอนเชา~2ชม. มีอาการออนเพลีย น้ําหนักลด4kg ใน 4เดือน ตรวจรางกาย T37.4 BPและ
                     B. HBS Ag , HCV Ag , Anti HAV                                                  P ปกติ พบinflammation ของ 2nd, 3rd Rt. MCP
         16. ผูปวยหญิง อายุ 46 ป ไมมีอาการผิดปกติใดๆ ตรวจรางกายปกติ ดื่มสุราเปนครั้งคราว      3rd, 4th Lt. PIP, both wrist, elbow, knee จงใหการวินิจฉัย
         ดื่มแตเบียร มากพอสมควร BMI 26 , waist circum 92 , BP normal , FBS impaired , AST                     A. SLE                      B. Rheumatoid arthritis
         elevated , ALT elevated (AST<ALT) ดื่ม alcohol ครั้งคราว มา 5 ป ขอใดคือสาเหตุสําคัญ                  C. Rheumatic fever          D. Undiff spondyloarthrepathy
         ที่ทําให liver enzyme ขึ้นในผูปวยรายนี้                                                             E. Ankylosing spondylitis
                     A. chronic viral hepatitis B. alcoholic fatty liver disease                    20. ชายปวด right ankle BMI สูง , FBS 123 , LDL 100 , HDL 50 ให colchicin แลวอาการ
                     C. autoimmune hepatitis D. non – alcoholic steatosis                           ดีขึ้น ทําอยางไรตอ
                     E. non – alcoholic steatohepatitis                                                         A. exercise                          B. allopurinol
                                                                                                                C. gemfibrocil                       D. simvastatin
         <<<<< Rheumatology >>>>>
         17. หญิงอายุ 25 ป มีอาการปวดขอมือ ปวดขอนิ้วมา 1 สัปดาหกอน มีผื่นผิวหนัง malar         <<<<< Nephrology >>>>>
         rash, oral ulcer U/A RBC 10-20/HF, few granular casts, proteinuria
                    จะสง Investigation อะไรตอ

Comprehensive - Medicine                                                          หนาที่ 4             Comprehensive - Medicine                                                            หนาที่ 5
21. ผูหญิง 30 ป ปสสาวะเปนสีน้ําลางเนื้อ 3 วันกอน มีไข ไอ เจ็บคอ ซื้อยามากินเอง PE            C. Vit B12 1.2 mcg/day               D. Cholinesterase inhibitor
         T= 38 no edema, BP 150/70 mmHg, mild pharynx injection, lung clear, cvs normal U/A                 E. Benzodiazepine
         RBC 100/HPF, WBC 5-10/HPF จงวินิจฉัย                                                       25. ชาย 25 ป โรงงานเทียน มีสวนประกอบethanol สารเรงการแข็งตัวของเทียน ทะเลาะ
                    A. IgA nephropathy                                                              กับหัวหนา กินอะไรเขาไป แลวหมดสติ คิดวาเกิดจากอะไร
                    B. PSGN                                                                                  A. Alcohol intoxication              B. Alcoholic psychosis
                    C. Ureteric stone                                                                        C. Hypoglycemia                       D. Hyporkalemia metabolic acidosis
                    D. Acute pyelonephritis
                    E. Acute interstitial nephritis
                                                                                                    <<<<< Endocrinology&Metabolic >>>>>
         22. ชาย 60 ปมาดวยปวดเอวซาย fever with chill ปสสาวะสีเขม
                                                                                                    26. ชายอายุ 50 ป มีอาการมึนศีรษะ หนักทายทอยมา 1 เดือน ตรวจรางกาย BP 170/70
         PE: Lt CVA tenderness, UA : WBC 50-100/HP , RBC 5-10/HF
                                                                                                    mmHg ระบบอื่นๆปกติ ผูปวยเปนความดันโลหิตสูงมา 1 ป ไมไดรักษาใดๆ FBS 80
         CBC : leukocytosis ( N=80, L=20 )
                                                                                                    mg/dL BUN 20 Cr 1.0 electrolyte อื่นๆปกติ ควรใหรักษาผูปวยอยางไร
         จะสงตรวจทางหองปฏิบัติการที่เหมาะสมตอไปอยางไร
                                                                                                                A. HCTZ 25 mg OD                      B. Enalapril 10 mg bid
                    A. U/S KUB                      B. IVP
                                                                                                                C. Nifedipine 5 mg tid                D. Lorsartan 100 mg OD
                    C. Retrogradeurethrography
                                                                                                                E. Propanolol 10 mg tid
         23. On NSAID (no cast)
                                                                                                    27. ผูปวยชายอายุ 47ป blood sugar 250 , HDL 30 , Cholesterol 193 , LDL 130 , TG 180
                    A. AIN                          B. acute tubular necrosis
                                                                                                    ผูปวยมี rish ตอ artherosclerosis ระดับใด และ goal ในการควบคุมเปนอยางไร
                                                                                                                A. very high risk , LDL<70
         <<<<< Toxicology >>>>>                                                                                 B. very high risk , LDL<70 , HDL>40
         24. ผูปวยชายอายุ 45 ป มีประวัติ heavy alcohol drinking เคยมีอาการสับสนเมื่อขณะลด                    C. very high risk , LDL<70 , HDL>40 , TG<150
         ปริมาณการดื่มเมื่อ 6 เดือนกอน หลังจากนั้นมีอาการหลงลืม อารมณเฉยเมย พูดในสิ่งที่                      D. high risk , LDL<100
         ไมเคยเกิดขึ้น รักษา                                                                                   E. high risk , LDL < 100 , HDL > 40
                    A. Vit E 1,000 IU/day                 B. Vit B1 100 mg/day


Comprehensive - Medicine                                                          หนาที่ 6             Comprehensive - Medicine                                                            หนาที่ 7
28. ชายอายุ 35 ป มีอาการเหนื่อยงาย ใจสั่น ตรวจรางกายพบวามี thyroid nodule ตรวจทาง         33. หญิง14 ป มีกอนที่คอเห็นไดชัด ไมเจ็บ ไมมีใจสั่น ไมมีน้ําหนักลด PE : generalized
         หองปฏิบัติการ TFT พบ T3, T4 สูง TSH ต่ํา จะสงตรวจเพิ่มเติมอะไรตอ                           enlargement of thyroid gland not tender , TFT euthyroid Mx ที่เหมาะสม
                    A. Thyroid scan                          B. Thyroid antibody                                 A. thyroid scan                         B. FNA
                    C. CT                                    D. MRI                                              C. Anti-microsomal Anti thyroglobulin
                    E. FNA                                                                                       D.ให thyroid supplement ไปกอน
         29. thyroid nodule : T4 ต่ํา TSH สูง จะ confirm ดวยอะไรกอน                                            E. subtotal thyroidectomy
                    A. thyroid scan                          B. FNA                                    34. แฟนพาผูปวยหญิงมาสงรพ. เปน DM on mixtard 70/30 40 U แบงใหเชา เย็น ac
                    C. biopsy                                                                          วันนี้ไมไดกินขาวเย็นแตฉีด mixtard เทาเดิม ตรวจรางกาย PR 100 RR 18 ซึม NS :
         30. ผูปวยหญิงอายุ 30 ป มีอาการเหนื่อยงาย ใจสั่น ประมาณ hyperthyroid T4 สูง T3 สูง         response to pain , CBG 26 mg/dl , จง management ที่เหมาะสม
         TSH ลด, thyroid gland: diffuse enlargement                                                              A. NSS 1,000 ml/hr
                    A. Subacute thyroiditis                  B. Grave’s disease                                  B. RI 0.1 U/kg IV bolus + RI 0.1 U/kg/hr
                    C. Hashimoto thyroiditis                                                                     C. 5%D/N/2 80 ml/hr
         31. ชาย 25 ป หิวน้ํา ฉี่บอย 3 สัปดาห มี deep breathing pattern มี BS 300, Na 130, K 2.5,             D. 50% glucose 50 ml IV push
         Ketone 8, BUN 10 initial Mx คือ?                                                              35. ชายสุขภาพดี FBS 400 ควรแนะนํา
                    A. NSS                                   B. RI                                               A. exercise
                    C. K after adequate renal function                                                           B.ใหยาเบาหวาน
                    D. K2PO4 after adequeate renal function                                            36. หญิงวัยกลางคน อาการ clinical Hyperthyroid ใจสั่นเหนื่อยงาย thyroid diffuse
         32. ผูปวยหญิงอายุ 60 ป U/D DM type II BW 80 kg, Ht 160 cm ตรวจ FBS ได 180 mg/dl           enlarge with thyroid bruit Lab T4 สูง T3สูง TSHต่ํา
         จะใหการรักษาอยางไร                                                                          การรักษาเบื้องตน
                    A. metformin                             B. glinide                                          A. PTU                                  B. B blocker
                    C. sulfonylureas                         D. alpha-glucosidase inhibitor                      C. Methimazole                          D. Lugol solution
                    E. glutadienediane
                                                                                                       <<<<< Dermatology >>>>>

Comprehensive - Medicine                                                            หนาที่ 8               Comprehensive - Medicine                                                              หนาที่ 9
37. ผูหญิงอายุ 60 ป มีผื่นคันที่ขาหนีบทั้ง 2 ขาง ทายาแลวไมดีขึ้น ผื่นลาย ตรวจรางกายพบ              E. bullous impetigo
         bilateral well defined erythematous patches surrounded with erythematous papule จงให         41. หญิงมีไข มีผื่นที่หนา ตอมาลามไปแขน PE: multiple erythematous papules
         การรักษา                                                                                                 A. Rubella
                    A. Mild potent topical steroid             B. Povidine iodine                      หญิง 18 ป มีตุมขึ้นที่หนา เปนๆ หายๆ 1 ป ไมคัน PE: open and closed comedones at
                    C. Calcineurin inhibitor                   D. Ketoconazole                         forehead and cheeks จงใหการรักษา
         38. Group of vesicles on erythematous base ที่รมฝปากบนซายสงตรวจ
                                                             ิ                                                    A. 0.25 % tretinoic acid                B. 1% clindamycin solution
                    A. KOH preparation                                                                            C. 4% erythromycin gel                  D. Oral tretinoin
                    B. Fresh preparation                                                               42. หญิงสามสิบป มีไขต่ํา ๆ มาสามสัปดาห น้ําหนักลดลง หา กิโลกรัมในหนึ่งถึงสอง
                    C. Wright stain                                                                    เดือน มีผื่นแดงกดเจ็บขึ้นที่หนาแขง PE: multiple discrete erythematous non ulcerative
                    D. Indian ink preparation                                                          papule at both shins จงสง Ix for Dx
         39. ผูปวยชายอายุ 60 ป มีตุมแดงเจ็บมานาน 1 เดือน PE: multiple tender erythematous                     A.CBC                                   B.U/A
         nodules at trunk, arms, legs, multiple infiltrative plaques with ill-defined border at ear               C.CXR                                   D.stool exam
         lobes,back, non caseating granuloma IX สง slit scrape ที่ ear lobe, nasal พบ AF bacilli                 E.?? (ไมมี skin biopsy)
         ถาม Dx?                                                                                       43. ชายมีปวดเมื่อยตามตัว มีผื่นใบหนาลําตัว แขนmultiple discrete vesicle pustule
                    A. Tuberculous leprosy                                                             hemorrhagic crust ที่ขา
                    B. Lepromatous leprosy with type I reaction                                                   A. chicken pox
                    C. Lepromatous leprosy with type II reaction
                    D. Vasculitis                                                                      <<<<< Neurology >>>>>
                    E. Cutaneous Tuberculous                                                           44. หญิงอายุประมาณ 30 ป ปวดหัวมาประมาณ 3 เดือน เปนมากตอนตื่นนอน ถาปวดมาก
         40. หญิง6 yr U/D lymphoma ไขสูง มีผื่นบริเวณลําตัว PE groups of vesicle pustule ตาม          จะมีอาการตามัวรวมดวย มี vertigo, N/V รวม ตรวจรางกายระบบอื่นปกติดี no
         L4 distribution c demarcated erythematous skin adjacent to the vesicle Dx                     neurological deficit จงใหการตรวจรักษาที่เหมาะสม
                    A. disseminated varicella                  B. disseminated herpes                           A. treat migraine
                    C. varicella c cellulites                  D. herpes zoster c erysipelas                    B. treat tension headache

Comprehensive - Medicine                                                          หนาที่ 10                Comprehensive - Medicine                                                            หนาที่ 11
C. LP                                                                        <<<<< Critical care >>>>>
                   D. CT                                                                        49. ชายอายุ 70 ป เปน DM นาน on insulin 70:30 เชา 40 unit เย็น 20 unit เชานี้กินขาว
                   E. MRI include cervical spine                                                ไมได แตไมไดลดยาฉีด ตอมามีอาการซึมลงจึงมาโรงพยาบาล เจาะ DTX = 23 mg/dl จง
         45. หญิง 30 ป มีอาการปวดหัวมากตอนเชา 30 นาที มีเวียนหัว สัมพันธกับกิจกรรมวิ่ง ควร   ใหการรักษา
         สง investigation                                                                                 A. 50% glucose 50 cc. iv stat
                   A. LP                                 B. CT                                             B. Draw blood sample สง confirm blood sugar
                   C. MRI                                D. Myelogram                                      C. 10% D/W iv drip
         46. ชาย อายุ 20 ป เหมอลอย แขนขาขวาเกร็งกระตุก 2-3 นาที กินยา phenytoin 300mg/d                  D. สง lab………..
         มานาน 5 ป ไดรับการวินิจฉัยเปน cysticercosis หยุดยาเองมา 1 เดือน จงใหการรักษา       50. ผูปวยอายุ 70 ป มี CA colon with brain&lung metastasis Dxเมื่อ 3 เดือนกอน ญาติ
                   A. phenytoin 300 mg/d w/o loading dose                                       ยอมรับได ไม CPR วันนี้ผูปวยซึมลง กินไดลดลง BP drop E1V1M1 จะใหการดูแล
                   B. phenytoin 300 mg/d with oral phenytoin loading dose                       อยางไร
                   C. phenytoin 300 mg/d with iv phenytoin loading dose                                    A. ET tube                            B. IV 1000 ml rate 80ml/hr
                   D. phenytoin 300 mg/d with diazepam loading dose                                        C. NG tube feeding -3 meals           D. ET tube then suction
                   E. phenytoin 300 mg/d with clobazam 10 mg/d                                             E. Admit
         47. ชาย 17 ป แขนขาออนแรง เคยเปนมากอนแลวกอนหนานี้ เมื่อ 3-4 WK กอน เมื่อคืน     51. ผูปวย CA colon with brain and lung metastasis ญาติทราบพยากรณโรคดี ผูปวยมี
         กนทํางานหนัก และกินอิ่มมาก Dx                                                         อาการซึมลง เหนื่อย เสมหะมาก กินไมคอยได มาร.พ. จะทําอยางไร
                   A. hypokalemic periodic paralysis                                                       A. ใส ET tube + suction
         48. ญ 60 ป ปวดศีรษะรุนแรง คลื่นไสอาเจียน แขนขาขวาออนแรง ตรวจได motor gr. 0/V                  B. admit และใหญาติเฝาตลอดเวลา
         BP 190/100 mmHg นาจะเปนอะไร                                                                    C. Oxigen mask with bag
                   A.venous sinus thrombosis             B.basal gg. hemorrhage                            D. salbutamol NB
                   C. Acute subarachnoid hemorrhage D. Acute MCA occlusion                      52. ผูปวยชายอายุ 60 ป BP 140/90mmHg มาทํา Exploratory Laparotomy เพื่อรักษา
                   E. frontal lobe hemorrhage                                                   rupture appendicitis หลังจากผาตัด ผูปวยมีอาการหนามืด เปนลมเวลาลุก PE: T 38.5 C,
                                                                                                BP 90/70 mmHg, PR 110/min, RR 30/min

Comprehensive - Medicine                                                     หนาที่ 12             Comprehensive - Medicine                                                              หนาที่ 13
จงใหการรักษาในผูปวยรายนี้                                                                         C. โอกาสเกิด rheumatic fever เทาคนปกติ
              A. ตรวจ Urine spec. gr. เพื่อพิจารณาเลือกใหสารน้ํา                                  57. คนไข heat stroke แมพามาหาหมอ เนื่องจากเปนลมบอย มักเปนตอนเขาแถวตอนเชา
              B. ให 5% Dextose in 0.45% NSS 100 ml/hr                                             ที่ รร. โดยเฉพาะวันที่อากาศรอน เปนจากเหตุใดมากที่สุด
              C. ให 0.9% NSS 200 ml/hr แลวประเมินซ้ํา                                                       A. vasovagal                        B. orthostatic hypotension
         53. ผูปวยกินปูทะเล,แมงดาทะเล ตอมา 1ชม. เกิด n/v กลามเนื้อออนแรง หายใจชา หนามืด                C. cardiogenic syncope
         เปนลม                                                                                    58. Septicshock ถามวาเกิดจากกลไกลใด
                    A. endotracheal intubation          B. adrenaline IM                                      a. ลดpreload                        b. เพิ่มafter load
                  C. bronchodilator nebulizer           D. NSS load                                           c. ลด sympathetic tone              d. ลด hemoconcentration
                  E. Lavage and activated charcoal                                                            e. ลด cardiac contract
         54. ผูปวยหยุดหายใจ pumpขึ้น ไดVF ทําไงตอ
                                                                                                   59. สงสัย IE ฟงได Pansystolic murmur Grade 3/6 at Apex , Diastolic murmur Grade 2/6
                    A. defibแลวpumpตอ
                                                                                                   at Apex นาจะเกิดจาก valvular lesion ใด
                                                                                                            a. MR                                 b. MR with relative MS
         <<<< Cardiology >>>>
                                                                                                            c. AS
         55. ผูปวยหญิง มีประวัติ CA breast ไดทํา modified radical mastectomy ได CMT 6 course
         ตอมามีอาการหอบเหนื่อยมา 2 สัปดาห JVP up to mandible PE. ฟง korokoff ไมชัดที่ 100
                                                                                                   <<<<< RS >>>>>
         mmHg มีๆหายๆ ที่ 80 mmHg จึงฟงไดชัด คิดวาเขากับโรคโดมากที่สุด
                                                                                                   60. ผูปวย CA lung with brain metastasis (Stage IV) admit อยูเพื่อให CMT หลังใหยาครบ
                    A. congestive heart failure B. cardiac temponade
                                                                                                   1 วัน ผูปวยมีอาการหอบเหนื่อย, O2 sat 90% ตรวจรางกายปกติ, CXR: no significant
                    D. constrictive pericarditis
                                                                                                   change from previous CXR ควรสง investigation ใดเพื่อใหไดประโยชน
         56. หญิงวัยกลางคนมีอาการเหนื่อยมากขึ้น pulse เร็ว พบ HR total irregularity ,loud S1,
                                                                                                               A. D-dimer                            B. ABG
         palpable P2,opening snap +, diastolic rumbing murmur gr II at apex lung clear
                                                                                                               C. CTA for pulmonary artery
                    A. มีโอกาสเกิด embolic stroke มากกวา normal 15-20 เทา
                                                                                                   61. Acute asthmatic attack Moderate distress ได O2 + salbutamol NB
                    B. โอกาสเกิด IE เทาคนปกติ
                                                                                                   ผูปวยควรไดอะไรเพิ่มเติม

Comprehensive - Medicine                                                       หนาที่ 14              Comprehensive - Medicine                                                               หนาที่ 15
A. Terbutaline SC                     B. Ipratropium NB                                 C. pulmonary embolism with pulmonary infarction
                    C. Theophyline IV                     D. Dexamethasone IV                               D. pulmonary TB
                    E. Beclomethasone NB                                                                    E. lung cancer
         62. ชาย 65 ป HT หอบเหนื่อย แพทยสง CXR สงสัย Lt pleural effusion ควรสงทาไหน
                    A. lt lateral decubitus               B. rt lateral decubitus                  <<<<< Infectious >>>>>
                    C. lordortic view                                                              66. ผูปวยชายอาชีพกรรมกร อายุ 30 ป มีไขสูงมา 4 วัน คลื่นไสอาเจียนไมถายเหลว และ
         63. ผูปวยใสมี hydropneumothorax ใส ICD ไดเลือด 400 ml ตอมา dyspnea and              ไมมีอาการอยางอื่น ยกเวน คัดจมูก น้ํามูกไหล ไอ Temp 39 C , pharynx injected , tonsil
         crepitation มี pink frothy sputum Dx                                                      enlarged , no jaundice , Liver just palpable mild tenderness จะสงตรวจอะไรเพื่อการ
                    A.reperfusion pulmonary edema         B.pneumonia                              วินิจฉัย
                    C.trauma from ICD                     D.Atelectasis                                       A. CBC                                  B. H/C
         64. ผูปวยชาย 20 ป ทํางานโรงงานพนสีรถยนต มีอาการเหนื่อยหอบ หายใจลําบาก มา                        C. LFT                                  D. Ab for dengue virus infection
         นาน 2 ป ตรวจการทํางานของปอดเปน restrictive lung disease ทานจะใหคําแนะนําแก                      E. Ab for Rickettsia infection
         ผูปวยรายนี้อยางไร                                                                      67. ชายอายุ 25 ป ถายอุจจาระเหลว 2 วัน วันละ 10 ครั้ง ไมปวดทอง อาเจียนวันละ 2-3
                    A. ยายงานหรือเปลี่ยนตําแหนง                                                  ครั้ง PE: T37c, P 110/min, BP 90/60mmHg Ix: CBC Hct 48%, Plt 120,000/mm3, WBC
                    B. ใชอุปกรณปองกันสวนบุคคลขณะทําการพนสี                                    13,000/mm3 (N 80%, L 20%), BUN 60, Cr 2, Na 142, K 2.8, HCO3 14, Cl 90 ถาม Dx?
                    C. เปลี่ยนเปนน้ํายาพนสีใหม                                                             A. Vibrio                               B. Amoebiasis
                    D. จัดหาพัดลมระบายอากาศ                                                                   C. Capillaria                           D. Salmonella
         65. ชายอายุ 60 ป U/D severe COPD , 3 วันกอน มีไข เสมหะสีเหลือง PE: V/S temp 38.2                  E. isospora
         C , RR 28 , BP 140/90 , O2Sat 80% , ตรวจ Lung ได fine crepitation RLL ตรวจ chest X-      68. หญิง 35 ป มาดวยมีไข ปวดหัว 10 วันกอนมารพ. T 38 , PR 100/min, BP 100/60,
         ray พบ alveolar infiltration at RLL , generalized wheezing both lung จงใหการวินิฉัยที่   conjunctival injection, liver just palpable, spleen ไมโต , generalized MP rash at trunk ,
         นาจะเปนไปไดมากที่สุด                                                                   extremities CBC : Hct 42 % , WBC 7,000 MCV 60 , L 40%, Plt 80,000 diagnosis, total
                    A. COPD exacerbation                                                           biliruin 15
                    B. RLL pneumonia                                                                          A. Dengue fever                         B.Scrub typhus

Comprehensive - Medicine                                                       หนาที่ 16               Comprehensive - Medicine                                                                หนาที่ 17
C. Leptospirosis                        D. Typhoid fever                                       E. เชื้อที่ติดเปนเชื้อรุนแรง
                  E. Viral hepatitis                                                                73. คนไขมี dengue fever มีอาการของ rhonchi คิดวาเกิดจากอะไร
       69. หญิง 60 ป U/D DM มีผื่นแดงที่หลังคอ มีตุมหนองหลายตุมที่บริเวณผื่นแดง Dx?                           A. plasma leakage                       B. Hypovolumic shock
                  A. cabuncle                             B. cellulitis                             74. ผูปวยชายอายุ 23 ป มีผื่นขึ้นมา 1 สัปดาห 3เดือนกอน มีเพศสัมพันธกับเพื่อนสาวที่
                  C. erysipelas                           D. ecthyma                                เพิ่งรูจักกัน อูวว ไมมีแผลที่อวัยวะเพศ PE: ผื่น erythematous บริเวณฝามือฝาเทา และแขน
       70. ผูปวย 17 ป มีแขนขาออนแรง 3 วัน เคยมีประวัติ IVDU มากอน ออนแรง lower grade          ขาทั้ง 2 ขาง จะรักษาอยางไร
       3/5, decrease pain sensation, hyporeflexia, loose sphincter tone, EYE : exudates and                      A. Ceftriaxone 250mg IM 1 dose
       hemorrhage at retina นาจะเกิดจากเชื้อใด                                                                  B. Benzathine Penicillin G 2.4M unit IM 1 dose
                  A. CMV                                  B. EBV                                                 C. Benzathine Penicillin G 2.4M unit IM 1dost/week , *3 weeks
                  C. HIV                                  D. S.pneumoniae                                        D. Prednisoline
                  E. S.aureus                                                                                    E. Oral Tetracycline 250mg qid 15 days
       71. ชายมีตุมน้ํา(แผลและตุมน้ํา)ที่อวัยวะเพศ เปน ๆ หาย ๆ มา 3-4 ป ชวง 2-3 เดือนเปนถี่   75. ชายอายุ 30 ป ตรวจพบ Non painful vertical whitish plague along the lateral long
       ขึ้น ใหการรักษา                                                                             border of tongue, the lesion is adherent to tongue. รอยโรคดังกลาวนาจะเกิดจากเชื้อใด
                  A. Acyclovir (200) วันละ 5 เม็ก 6 เดือน                                           มากที่สุด
                  B. Acyclovir (400) วันละ 5 เม็ก 6 เดือน                                                        A. EBV                                  B. Staphylococcus aureus
                  C. Acyclovir (800) วันละ. 5 ครั้ง 5 วัน                                                        C. Streptococcus pneumoniae             D. Pox virus
                  D. Acyclovir cream ทาจนกวาจะหาย                                                               E. Papilloma virus
                  E. พญายอครีมทา                                                                    76. ชาย 30 ป มามโต 1 cm ตับโต 1 cm below costal margin มีไขมา 3 สัปดาห
       72. ชายอายุ 25ป เปนโรคลมชัก กินphenytoin tid มา5ป 1วันกอนมีอาการไอ ไข เจ็บคอ ไป                      A. dengue hemorrhagic fever             B. salmonella
       คลินิกไดATBมากิน ตอมาอาการหนัก ไขขึ้นสูง หนาวสั่น ไปร.พ. แพทยบอกติดเชื้อใน                            C. murine typhus                        D. lymphoma
       กระแสเลือด เกิดจากสาเหตุใด                                                                   77. หญิง อายุ 40ป ติดเชื้อCryptococcus แตHIV negative จะตรวจอะไรเพิ่ม
                  A. ATB ไมพอ                            B. โรคลมชักเสี่ยงpneumoniaจากสําลัก                    A. IgG                 B. IgM           C. IgA
                  C. ATB ผิดชนิด                          D. ยาที่ใหทําใหเม็ดเลือดขาวต่ํา                      D. CD4                 E.CD8

Comprehensive - Medicine                                                           หนาที่ 18               Comprehensive - Medicine                                                             หนาที่ 19
- - - - - - -- - - - - - - - - - 02 Pediatrics - - - - - - - - - - - - - - - -
          <<<<< Neonatology >>>>>
          78. เด็กทารกอายุ 12 ชม. มีอาการชักเกร็งกระตุก 2 นาที ระดับน้ําตาลในเลือด 29 mg/dL,
          น.น.แรกเกิด 2950 g, Ht 47 cm, HC 37 cm ตรวจรางกาย T 37 °C, P 120/min, RR
          40/min, BP 78/56 mmHg ตรวจรางกายอื่นๆพบ cleft palate, penis length 2.1 cm, urethral
          opening at ปลาย penis, testes ทั้ง 2 ขางอยูใน scrotal sac ถาม cause ของ hypoglycemia
          ในผูปวยรายนี้ที่เปนไปไดมากที่สุดคือ
                      A. Hyperinsulinism                     B. Gonadotropin deficiency
                      C. Growth hormone deficiency           D. Glycogen storage disease
                      E. Primary adrenal insufficiency                                                       A. Midgut volvulus                  B. Intussusception
          79. เด็กเพศหญิงคลอดครบกําหนด Hct 50% MB 9.6 mg/dL แรกคลอด น.น. 3000g กวาๆ                         C. NEC                              D. Meconium peritonitis
          ที่ 36 ชั่วโมงหลังคลอด น้ําหนัก 2820 กรัม กินแตนมแมอยางเดียว ศีรษะไม swelling ให    82. เด็กแรกเกิดหนัก 3000g ทํา newborn screening พบวา thyroid function test ที่ day 3
          กราฟ CPG jaundice ของเด็ก newborn มา ถามวาจะ manage อยางไร                             (หรือday5หวา) TSH=?=สูง T4=0.5=ต่ํา ควรทําอยางไร
                      A. ใหเด็กกลับบานได                  B. ใหเด็กกินนมแมมากขึ้น                       A. ให Levothyroxine                B. ทํา thyroid scan
                      C. phototherapy                        D. ให IV                                       C. เจาะ TSH & T4 ซ้ํา day14         D. เจาะ T3
                                                      o
          80. ทารก GA 34 wks BW 1850 g, T 38.5 C, BP ไม drop, abdomen: soft, hypotonia,           83. ทานไดรับมอบหมายใหดูแลทารกคลอดครบกําหนด มีปากแหวงเพดานโหว HC 31
          abdominal distention, capillary refill 2 sec หลัง septic W/U ทําอะไรตอ                  cm มีตาชิดและเล็ก systolic murmur grade 3/6 มือทั้งสองขางมีนิ้วเกิน ทานจะดูแลทารก
                      A. ให empirical ATB                   B. 0.9% NSS 10 cc/kg V push           รายนี้อยางไรกันละหนอ
                      C. Urinary catheter                                                                    A. บอกพอแมวาไมเปนอันตราย สามารถผาตัดรักษาได
          81. ทารกคลอด C/S GA 28 wks due to fetal distress APGAR 5,7 อยู ICU ตลอด 8 วัน มี                  B. แนะนํากระตุนพัฒนาการ และตรวจการไดยินตอนอายุ 1 ป
          อาการทองอืด ถายเปนมูกเลือด film abdomen ดังรูป Dx?                                              C. ปรึกษาศัลยตกแตงแกไข ปากแหวงเพดานโหว
                                                                                                             D. สง CT ดูสมองเพราะหัวเล็ก


Comprehensive - Pedriatrics                                                    หนาที่ 20              Comprehensive - Pedriatrics                                                         หนาที่ 21
E. บอกพอแมวาพยากรณโรคในทารกไมดี                                            88. เด็ก 1 วันกอน ไอ 1 ชั่วโมงกอน ไอมากขึ้น trachea shift to the Rt, mediastinum shift
        84. เด็กอายุ 30 สัปดาห มีฟนขึ้น 2 ซี่ลางดานหนา #71, #81 ฟนโยก มารดาพามาพบแพทย       to the Rt, decreased breath sounds Rt, CXR ทึบที่ดานขวา ควรสงตรวจอะไรเพิ่มเติมดวย
        จงใหการวินิจฉัย และ คิดวาเกิดจากความผิดปกติในขั้นตอนใด                                             A. Bronchoscopy                      B. CXR Rt lateral decubitus
                   A. Neonatal teeth / Initiation phase                                                     C. CT
                   B. Natal teeth / Initiation phase
                   C. Neonatal teeth / Proliferative phase
                                                                                                   <<<<< Infectious >>>>>
                   D. Natal teeth / Proliferative phase
                   E. Supranumeral teeth / Initiation phase                                        89. เด็กชายอายุ 6 ป ไขต่ําๆ ปวดทอง อาเจียน 2 wks PE เหลืองเล็กนอย ตับโตกดเจ็บ
        85. เด็ก preterm GA 34 wks BW1650 g, APGAR 8,9 ที่1,5 min แรกคลอดมีปญหาเรื่อง             LAB: SGOT 1500 U/L, SGPT 2500 U/L, anti-HCV neg, anti-HAV IgM pos, anti-HAV
        การหายใจ feed via OG tube ตั้งแตวันที่ 2 วันนี้อายุ 16 วัน มี apnea 2 times in 30 min,    IgG neg, anti-VCA IgG pos, anti-VCA IgM neg, HBs Ag neg, anti-HBc pos, anti-HBs
        tactile stimulation ได BT 36.3oC, HR186/min, BP 86/56 mmHg, RR 56/min, inactive,          pos, mycoplasma titer 1:40 นาจะติดเชื้อใด
        hypotonia, decreased bowel sounds, distension abdomen หลังจากทํา septic work up แลว                 A. HAV                                   B. HBV
        ควรทําอะไรตอเปนอันดับแรก                                                                           C. HCV                                   D. EBV
                   A. Empirical ATB                        B. NSS drip 10 cc/kg                              E. Mycoplasma
                   C. Oxygen hood                          D. Foley catheterization ติดตาม urine   90. เด็กชายอายุ 6 wks ไอเปนชุดๆจนหนาเขียวที่รอบปากและปลายมือ บางครั้งอาเจียน
                   E. IVIg                                                                         หลังไอ เวลาที่ไมไอกินนมไดตามปกติ ไมเขียว มีประวัติมารดาไอเปนชุดๆมา 2 wks
        86. เด็กคลอดติดไหล หลังคลอดพบแขนซาย flaccid, Lt lung decreased breath sounds and         ตรวจรางกาย T 39oC, RR 18/min, no dyspnea, normal breath sounds จงวินิจฉัย
        movement สาเหตุเกิดจาก                                                                               A. Acute bronchitis due to RSV
                   A. phrenic nerve injury                 B. Brachial plexus injury                         B. Pneumonia due to Chlamydia trachomatis
        87. ทารกแรกเกิด แขนขวาไมขยับ ตอนคลอดไหลขวาติด ตองดึงใชแรงมาก ควรตรวจ                             C. Acute bronchitis due to Mycloplasma pneumoniae
        อะไรเพิ่มเติม                                                                                        D. Bronchiectasis due to Strep pneumo
                   A. CBC                                  B. Moro reflex                                    E. Pertussis due to Bordetella pertussis
                   C. Asymmetrical tonic neck reflex D. Parachute reflex

Comprehensive - Pedriatrics                                                      หนาที่ 22               Comprehensive - Pedriatrics                                                         หนาที่ 23
91. หญิงอายุ 15 ป ปวดเมื่อย 3 วัน เริ่มมีผื่นขึ้นที่ใบหนา ลําตัว, PE T=39oC, multiple                      C. Rabie
          posterior cervical LN ทั้ง 2 ขาง, mild tenderness, mild MP rash at face & trunk, mild            95. เด็กมีไขปวดหัวมา 10 วัน มี injected conjunctivitis, ไมมี eschar, platelet 85000/mcL,
          pharyngitis, CBC พบ Hct 43%, WBC 9500/mcL (N50%, L50%) plt 150000/mcL จง                          LFT???, Cr.1.0 จง Dx
          วินิจฉัย                                                                                                     A. Leptospirosis                      B. Scrub typhus
                     A. Streptococcal pharyngitis               B. Viral pharyngitis                                   C. Typhoid fever
                     C. Rubella                                 D. Dengue fever                             96. เด็กอายุ 3 ป เจ็บ+ฉี่แสบ PE พบ vulva บวมแดง, swab discharge C/S พบ
                     E. Infectious mononucleosis                                                            intracellular Gram negative diplococci จงใหการวินิจฉัย
          92. เด็ก (ไมไดบอกอายุ) มีอาการไข 4 วัน เจ็บคอ 5 วัน 2 วันกอนไปพบแพทยไดรับ                              A. UTI                                B. Vulvitis + UTI
          Amoxicillin อาการไมดีขึ้น วันนี้มีผื่นขึ้น PE T 39.0 oC, puffy eyelid, tonsil enlarged,                   C. Gonorrhea                            D. UTI+ gonorrhea with child abuse
          injected with exudate, MP rash at trunk and extremities, Hct 42%, WBC 18000 (N15%,
          L70%, Atypical Lymph 10%, Monocyte 5%) Plt 18000, พบ Downy cell ใน Peripheral
          Blood smear
                                                                                                            <<<<< Nutrition >>>>>
          จงใหการวินิจฉัย                                                                                  97. เด็กสงสัย CP แมปอนดวยขาว กระดูกหมู ไข วันละ 3-4 มื้อ มีบวมที่เขาขวา ปวดแดง
                     A. Streptococcal pharyngitis               B. Rheumatic heart disease                  ขอใดถูกตอง
                     C. Infectious mononucleosis                                                                      A. เลือดออกงายจาก platelet dysfunction
          93. เด็กอายุ 4 ป ติดเชื้อบอย ไข ทองเสีย มี oral thrush มี PCP ที่ lung แมไมมี HIV risk จะ             B. พบโรคหัวใจเนื่องจาก heart dysfunction
          สงตรวจอะไร                                                                                                 C. กินผักใบเขียวปองกัน
                     A. Ig                                      B. CD4, CD8, CD3, NK                                  D. มีการทํางานผิดปกติของตับ
                     C. Complement                              D. Phagocyte function
          94. เด็กอายุ 6 สัปดาห dyspnea, O2sat. 88%, both upper lung crepitation and rhonchi ,             <<<<< Development >>>>>
          ปากเด็กมี oral trush แมไมมีประวัติ HIV คิดถึงอะไร                                               98. เด็กอายุ 6 ป ตอนนี้เด็กอยูอนุบาล เวลาทําการบานตองใหแมชวย ซนตามวัย เขากับ
          เด็กชาย 12 ป ชักเกร็ง รูสึกตัว อาปากไมสุด แสยะยิ้ม ถาเสียงดัง แสงจาเปนมากขึ้น Dx           เพื่อนได วันนี้แมพามาฉีด vaccine แมกังวลเรื่องลูกจะเรียนไดหรือไม จะประเมินอยางไร
                     A. Tetanus                      B. Epilepsy                                                       A. ใหเด็กเลาเรื่องที่โรงเรียน


Comprehensive - Pedriatrics                                                            หนาที่ 24                Comprehensive - Pedriatrics                                                             หนาที่ 25
B. ประเมิน attention span จําของสามอยาง                                                    C. Cystitis                          D. IgA nephropathy
                    C. ใหเด็กวาดรูปคน+รูปเรขาคณิต                                                              E. Postinfectious glomerulonephritis
                    D. ขอดูสมุดบันทึกการเรียนจากครู                                                   102. เด็กหญิงอายุ 5 ป มารพ.ดวยทองเสียและอาเจียนมาก ปสสาวะไมออก มา 12 ชม. ได
                    E. ซักประวัติเรื่องเรียนกับแม                                                    IV fluid 20 ml/kg ตรวจ serum K 10 mmol/L, Cr 2.5 mg/dL, EKG wide QRS complex
          99. เด็กชายอายุ 6 ป พัฒนาการชา พูดเปนคําๆ พอฟงรูเรื่อง เดินไมคอยได ใชชอนกินขาว   ขอใดเปนสาเหตุของ EKG ที่ผิดปกตินี้
          ไมได ไปกองน้ําเองไมได ถาทําการทดสอบพัฒนาการของเด็กอายุนอยกวา 6 ป จะได IQ                    A. HyperNa                           B. HyperK
          เทาไร                                                                                                C. HyperCa                           D. HyperMg
                    A. <25                               B. 55-70                                               E. Hyperphosphatemia
                    C. 70-90                             D. 90-110
                    E. >110                                                                           <<<<< Allergy >>>>>
                                                                                                      103. ผูปวยเด็ก 30 min PTA ถูกผึ้งตอย 15 แผล มีปากบวม ชารอบปาก N/V หายใจหอบ
          <<<<< Nephrology >>>>>                                                                      เหนื่อย BP drop, wheezing คิดวาเกิดจากกลไกใด
          100. เด็กอายุ 6 ป ปสสาวะสีแดงมา 2 วัน ปสสาวะออกนอยลง ไมเคยเปนมากอน ตรวจ                         A. Anaphylatic reaction                 B. Septic shock
          รางกาย BP 130/90mmHg, puffy eyelid, UA: sp.gr.1.015 protein1+ RBC100-200/HP                           C. Histamine release syndrome           D. Local reaction
          RBC cast 0-2/LP ไมไดรับการรักษา ปสสาวะออกเองไดดี ยุบบวมใน 4 วัน การตรวจทาง                         E. Mast cell ปลอยสารออกมา
          หองปฏิบัติใดนาจะเปนไปไดในผูปวยรายนี้                                                  104. เด็กหญิงอายุ 6 ป มีผื่นลมพิษทั่วตัว 2 hrs กอนมา รพ. เริ่มมีอาการหลังกินกุง 15 นาที
                    A. Antinuclear Ab titer 1:2560         B. Anti-streptolysin O titer 100 IU/mL     ลมพิษขึ้นหนาและตัว BP 75/40 mmHg, RR 40/min, pulse 110/mon, no wheezing, GI
                    C. Serum C3 33 mg/dL                   D. Urine Na 40 mmol/L                      increased bowel sounds
                    E. Urine protein to creatinine 5 mg/mg                                                       A. Direct release mediator from mast cell
          101. เด็กหญิงอายุ 10 ป มีอาการปสสาวะบอย และปสสาวะเปนสีแดง รวมกับมีไข ไอมา                       B. Complement deactivation
          2-3 วัน กอนหนานี้เคยเปน 1 ครั้งหายแลว ตรวจรางกายปกติ Urinalysis พบ pH 6.0, WBC                    C. Disturbance of arachidonic acid metabolism
          5-10/HP, RBCสูง รวมกับ dysmorphic RBC จงใหการวินิจฉัย                                                D. Type I hypersensitivity
                    A. Hypercalcuri                        B. Renal calculi                                      E. Eosinophil degranulation

Comprehensive - Pedriatrics                                                        หนาที่ 26              Comprehensive - Pedriatrics                                                             หนาที่ 27
105. เด็กหญิง asthma มา 5 ป มาดวยอาการหอบเหนื่อย P110/min, BP 140/70mmHg,            <<<<< Cardiology >>>>>
        rhonchi ทั้ง 2 ปอด ได O2 + sulbutamol NB ควรใหอะไรอีก                                108. เด็กหญิงอายุ 12 ป มาโรงพยาบาลดวยอาการใจสั่น ตรวจรางกายพบ T 37oC, P
                  A. IV Dexa                          B. IV terbutaline                        200/min, BP 100/70 mmHg ให EKG มาดังรูป จงใหการรักษา
                  C. Theophylline                     C. NB ipratropium
                  E. NB beclomethasone

        106. เด็กชายอายุ 8 ป มี VSD ใหเลือดกอนผาตัด ขณะไดเลือดไป 5 ml ผูปวยมีอาการหอบ
        เหนื่อย wheezing, BP ลดจาก 90/50 เปน 60/20 ไมมีไข หลังจากหยุดใหเลือดแลว ทานจะ
        ทําอะไรตอ
                  A. Adrenaline                         B. Antihistamine
                 C. Dexamethasone                    D. Respiratory support
                 E. ตรวจสอบเลือดที่ให                                                                   A. Adenosine 0.1 mg/kg IV                B. Digitalis 0.1 mg/kg IV
                                                                                                         C. Verapamil 0.01 mg/kg IV               D. DC synchronized cardioversion
        <<<<< Psychiatry >>>>>                                                                           E. CPR
        107. เด็กหญิงอายุ 7 ป มาดวยปวดทอง คลื่นไส ปสสาวะราดตอนกลางคืน กลางวันปกติ มี      109. เด็กอายุ 3 ป ไข อาเจียน 4วัน ตับโต มีจุดเลือดออก BP 90/70 mmHg, check จากอะไร
        อารมณหงุดหงิดฉุนเฉียว แขนขาออนแรง ตรวจรางกายพบวา T 38.5oC, BP 100/70                         A. Decreased H/S concentration           B. Decreased sympathetic activity
        mmHg, GA: inactive, resp.: dyspnea with fruity odor, Ix: hyponatremia, K4.5 จง                   C. Decreased cardiac contraction         D. Decreased preload
        investigation ที่เหมาะสม                                                                         E. Increased afterload
                  A. blood glucose                     B. blood lactose                        110. เด็กอายุ 7 ป มีไข เหนื่อย หอบ ปวดขอ, มีไข PR&RR ปกติ, ไมมีผล lung, CVS พบ
                  C. U/S whole abdomen                 D. CT brain                             systolic ejection murmur 2/6 ที่ apex กับ diastolic blowing murmur 2/6 ที่ apex บอกวา
                  E. UA                                                                        นาจะมีภาวะใด
                                                                                                         A. AR                                    B. PR


Comprehensive - Pedriatrics                                                   หนาที่ 28              Comprehensive - Pedriatrics                                                       หนาที่ 29
C. MR                                 D. MR with relative MS                     <<<<< Hematology >>>>>
                   E. AR กับ AS                                                                     114. เด็กอายุ 8 ขวบ มีอาการไขสูง, H/C พบ Gm neg rod, มี bleeding ไมหยุดที่ central
                                                                                                    line, เจาะเลือดพบ PT, aPTT, TT prolong ทั้งหมด ทานคิดวาจะเจอผล lab เปนอยางไร
                                                                                                               A. Fibrinogen ต่ํา D-dimer สูง       B. Fibrinogen ต่ํา D-dimer ต่ํา
          <<<<< RS >>>>>
                                                                                                               C. Fibrinogen สูง D-dimer สูง        D. fibrinogen สูง D-dimer ต่ํา
          111. สงสัย croup ใหการรักษา....
                                                                                                    115. เด็กอายุ 6 เดือน ชัก ซึม มีไข กระหมอมโปง, T 39°C, prolonged PT & aPTT, normal
          <<<<< GI >>>>>                                                                            TT, เด็กคนนี้มีความผิดปกติที่ Factor ใด
          112. ผูปวยเด็กมี URI symptoms, 2 วันตอมามีอาการทองเสีย อุจจาระเปนน้ํามากกวาเนื้อ               A. VII                                B. IX
          รองไหงอแง ตรวจรางกายพบ moderate dehydration ขอใดถูก                                              C. XI                                D. Prothrombin complex
                     A. ตรวจลมหายใจพบ H2, CH4                                                       116. ชายอายุ 7 ป Hx birth wt 1500g, เหนื่อย เพลีย ซีด, CBC พบ RBC normal with
                     B. ในลําไสจะพบ CO2, NH3 เพิ่มขึ้น                                             polychromasia นิดหนอย มามโต ตับไมโต DX
                     C. Stool exam พบ lactic acid เพิ่มขึ้น                                                    A. Congenital spherocytosis
                     D. Stool exam ไมพบ reducing substance                                                    B. Portal vein thrombosis
                     E. ใหดื่ม ORS อยางเดียวจนกวาจะหาย diarrhea
          113. เด็กชายอายุ 4 เดือน มีไขและอุจจาระเหลวมา 2 วัน อาเจียน อุจจาระเปนน้ําพุง เปนลม   <<<<< Endocrine >>>>>
          มาก ซึมจากการขาดน้ํา คิดวาติดเชื้ออะไร                                                   117. ผูปวยเด็กหญิงอายุ 14 ป มีอาการใจสั่น คอโต น้ําหนักลด, T3 & T4 สูง TSH ต่ํา,
                     A. Shigella                            B. Salmonella                           I131uptake สูง, thyroid scan: diffused enlargement, จงใหการวินิจฉัย
                     C. Cholera                             D. Rotavirus                                       A. Follicular CA                      B. …
                     E. Norovirus                                                                              C. Lymphoma                           D. Grave’s disease
                                                                                                    118. เด็กหญิงอายุหนึ่งขวบครึ่ง แมพามารับวัคซีน แพทยตรวจรางกายอื่นๆ ปกติ ยกเวน
                                                                                                    genitalia พบ
                                                                                                    - Rt.Labioscrotal fold มี mild rugae และ คลําได mass ขนาดประมาณ 1 cm
                                                                                                    - Lt Labioscrotal fold ปกติ คลําไดmass ขนาดประมาณ 1 cm ที่ low inguinal canal

Comprehensive - Pedriatrics                                                     หนาที่ 30              Comprehensive - Pedriatrics                                                         หนาที่ 31
- Phallus 1.5 cm length & มี urethral opening ที่ perineum                                    <<<<< Neurology >>>>>
          จงวินิจฉัย                                                                                    122. เด็กอายุ 8 เดือนมีไขมา 3 สัปดาห 1 สัปดาหกอนน้ําหนักลด 1 kg เด็กรองงอแง
                     A. Hypospadias with undescended testis                                             drowsy anterior frontanel บวม Meningeal irritation +ve PE : T 38.5 Bw 9 Kg
                     B. Male pseudohermaphrodite                                                        LP : Cell 300 (L 95%), Sugar 20(Blood 80), Protein 200
                     C. Female pseudohermaphrodite                                                      จงใหการวินิจฉัย
                     D. CAH                                                                                       A. Bacterial meningitis                B. Viral meningitis
          119. มารดาพา หญิง 13 ป มาดวย ปจด.ไมสม่ําเสมอ เริ่มมี ปจด.มาตั้งแตอายุ 12 ป 3 เดือน                 C. Fungal meningitis                  D. Eosinophilic meningitis
          มาเพียง 2 ครั้ง ครั้งละ 5-7 วัน กะปริดกะปรอย ไมมีปวดทองนอย ครั้งลาสุด 2 สัปดาห                     E. TB meningitis
          กอน PE: breast and pubic hair tanner stage 3, other unremarkable ทานจะใหคําแนะนํา          123. เด็กอายุ 15 เดือน ยาพามาปรึกษาเรื่องพัฒนาการชา เด็กพูดคุยได เลนไดดี นั่งเองได
          หรือรักษาอยางไร
                                                                                                        เกาะยืนไดบาง สังเกตวาเด็กไมคอยใชแขนขวา เวลายกตัวขาขวาจะเหยียดเกร็ง ตรวจ
                     A. ตรวจเลือด                            B. สงหาสูตินรีแพทย หาสาเหตุ
                     C. แนะนําวาเปนภาวะปกติที่พบได D. ใหยาพวกฮอรโมนปรับ ปจด.                       รางกาย เด็กเลนไดดี รูสึกตัวดี มี Lt hemiparesis, increased muscle tone, hyperreflexia
          120. เด็กชายอายุ 10 ป มาพบแพทยดวยเรื่องตัวเตี้ย โตชา แข็งแรงดี พอสูง 175 cm แมสูง       ของแขนซาย (โจทยมันใหขางซายขวามาอยางงี้เลยงงๆ) นึกถึงโรคใดมากที่สุด
          152 cm เด็กพัฒนาการปกติ ไมมีโรค ขณะนี้ BW at 10th percentile, Ht 125 cm (P3)                          A. Brain tumour                         B. Child neglect
          1 ปที่ผานมาโตขึ้น 5 cm, x-ray bone age = 8 yr                                                        C. Cerebral palsy                       D. Hydrocephalus
                     A. Familial short stature               B. Constitutional delayed growth                    E. Increased intracranial pressure
                     C. Malnutrition                         D. Cushing’s syndrome
                                                                                                        124. เด็ก 4 เดือน ซึม ไข ชัก มากอน สรุป meningitis อะไรสําคัญสุด
                     E. GH deficiency
                                                                                                                 A. LP                                   B. CBC
          121. เด็กอายุ 7 ป มีน้ําหนักขึ้น 7 kg ภายใน 5 เดือน สวนสูงเพิ่มขึ้น(จําตัวเลขไมได) ตรวจ
          รางกาย มี acne (signs of androgen เพิ่มขึ้น)                                                          C. CT brain
                     A. Late onset CAH                       B. Adrenal tumor                           125. ชายอายุ 10 ป ขา 2 ขางออนแรงและชา 3 วันเดินไมได อุจจาระปสสาวะไดปกติ
                   C. ACTH releasing tumor                                                              motor power: lower limbs grade 0, upper limbs grade 2, lesion อยูที่ใด
                                                                                                                 A. Brainstem                            B. Peripheral nerve

Comprehensive - Pedriatrics                                                        หนาที่ 32                Comprehensive - Pedriatrics                                                            หนาที่ 33
C. Muscle                              D. Spinal cord                                       A. Erythema infectiosum              B. Roseolar infantum
                 E. Cerebral hemisphere                                                             129. เด็กหญิงอายุ 17 วัน แมพามา รพ. เพราะมีผื่นมา 2 วัน PE: multiple discreted tiny
        126. หญิง 12 ป มาพบแพทยดวยอาการขาออนแรงทั้งสองขางมา 1 วัน โดยเริ่มมีอาการ              erythematous papules on forehead and neck, ตรวจ G/S + Wright’s stain ไมพบ bacteria
        ออนแรงตั้งแตเมื่อวาน ตื่นเชามาพบวาขาไมมีแรง ไมสามารถกลั้นปสสาวะอุจจาระได PE         or WBC คิดวาเปนไรนิ
        พบ hypotonia both lower extremities, motor power 0/V, areflexia, upper extremities                   A. Milia                             B. Miliaria rubra
        normal tone, lose of sensation below umbilicus, loose anal sphincter tone, Dx?                       C. Bullous pemphigoid                D. Erythema toxicum neonatorum
                 A. Transverse myelitis                 B. Poliomyelitis                                     E. Seborrheic dermatitis
                 C. Guillan-Barre syndrome              D. Cerebrovascular accident                 130. เด็กอายุ 7 ป เปนตุมสีเดียวกับเนื้อ ผิวขรุขระ ขึ้นกระจายตามแขน ขนาด 0.3-0.5 cm
                                                                                                    เปนมา 6 เดือน สังเกตวาพอเกาแลวตุมจะกระจายไปตามรอยเกา Dx?
        <<<<< Genetics >>>>>                                                                                 A. Melanocytic nevus                 B. Molluscum contagiosum
        127. เด็กแรกเกิด ใหลักษณะมาเปน Down’s syndrome สุดๆ แมอายุ 27 ป พออายุ 27 ป ขอ                C. Seborrehic keratosis              D. Chicken pox
        ใดถูก                                                                                                E. Verrucus valgaris
                 A. ไมตองตรวจ chromosome พอแม ถาตรวจลูกไดเปน trisomy
                 B. นาจะเกิดจาก translocation มากกวา trisomy
                 C. ถาเกิดจาก unbalanced translocation จะมีโอกาสในลูกคนตอไป 50%
                 D. ไมตองตรวจ chromosome เนื่องจากแมอายุนอยกวา 35 ป


        <<<<< Dermatology >>>>>
        128. เด็กอายุ 10 ป มีไขมา 3 วัน มีผื่นเปน confluent erythematous rash at both cheek มี
        reticulated …. ? Give diagnosis

Comprehensive - Pedriatrics                                                      หนาที่ 34               Comprehensive - Pedriatrics                                                       หนาที่ 35
- - - - - - - - - - - 03 Obstetrics & Gynecology - - - - - - - - - - -                     135. หญิงตั้งครรภ น้ําเดินมา 2 hr. กอนมาโรงพยาบาล ตรวจพบน้ําเดินจริง PV: station
                                                                                                     0, dilate 4 cm eff 90% หลังจากนั้น 2 ชั่วโมง station 1+, dilate 5 cm effacement 100%
          <<<<< Obstetrics >>>>>                                                                     มี earlydece จง management
          131. มาคลอด เฝาการคลอด PV ครั้งแรก effacement 90%, os 5 cm. อีก 1 ชั่วโมง                            A. Drip oxytocin                     B. C/S because arrest of dilatation
          effacement 100%, os 7 cm ทําไง                                                                        C. C/S station +1                    D. Amnioinfusn
                    ไมมี choice มาใหอะ                                                            136. หญิง 20ป G1P0 GA38wk มาดวยปวดครรภสม่ําเสมอ 3-5นาที นานครั้งละ 60วิ ตรวจ
          132. หญิงอายุ 33 ป ตั้งครรภ GA 33 wk. เด็กในทองไมไดดิ้นมา 6 wk u/s ไมเจอheart beat
                                                                                                     พบ dilate 3 cm, efface 90% แพทยทําการเจาะถุงน้ําคร่ําเรงคลอด และใหepidural infusion
          ตรวจพบมี Hct 34%, plt 70,000, PT17, PTT44, TT22 จะใหอะไร
                                                                                                     คุมอาการปวด 1ชม. ตอมา PV: dilate 5cm eff100% ทําอยางไรตอ
                    A. cryoprecipitate                     B. FFP
                                                                                                                A. ใหoxytocinเรงคลอด เพราะprotract active phase
                    C. Traxanemic acid                     D. low dose heparin
                                                                                                                B. observeตอ การคลอดเปนไปตามปกติ
                    E. Plt. concentration
                                                                                                                C. หยุด epidural infusion ใหปากมดลูกขยายไดดีขึ้น
          133. หญิงมาฝากครรภคิดวาตั้งครรภ 26 wks มากฝากครรภครั้งแรก F/U พบวาครรภมี
                                                                                                                D. เริ่มเบงได
          ขนาดเล็กกวา แพทยตรวจ GA นอยไป 5wk ขอใดไมใชสาเหตุของ IUGR
                                                                                                                E. C/S
                    A. จําปจดผิด                           B. Thalassemia
                                                                                                     137. หญิง ตั้งครรภ 24 สัปดาห มาดวยพบแพทยดวยอาการซึม ซีด สับสน ตรวจ Lab Hb 6,
                    C. แมติดเชื้อ                         D. GDM class B1
                                                                                                     WBC 12000 (Neutrophil 35%, Lymphocyte 65%) Platelet 15000, Coomb test- negative,
                    E. chromosome abnormality
                                                                                                     BUN 60, Cr 3.5, LDH 3000 Peripheral Blood smear- Schistocyte 2+
          134. หญิงตั้งครรภแรก GA 32 wk เมื่อ 2 hr กอน มีเลือดออกทางชองคลอดปริมาณมาก
                                                                                                     จงใหการวินิจฉัย
          ผาอนามัย 7-8 ผืนชุม มีทองแข็งเปนพักๆ มาตรวจพบ mildly pale VS stable FHR 140 มี
                                                                                                                A. Preecclampsia
          contraction ทุก 5 นาที ตรวจ PV มีเลือดคางอยู 20 ml ในชองคลอด เลือดหยุดไหล แลวเคย
                                                                                                                B. Acute Disseminated intravascular coagulopathy
          มีประวัติเลือดออกทางชองคลอดเมื่อ 1 เดือนกอนและ 2 wk กอน
                                                                                                                C. HELLP syndrome
                    A. tocolytic drugs                     B. oxytocin augmentation
                                                                                                                D. ITP
                    C. PV ประเมิน BISHOP score             D. Abdominal ultrasound
                                                                                                                E. TTP
                                                                                                     138. หญิงเพิ่งคลอดลูก day3 ตอนนี้มีfluctuant nodule ที่breast ซาย ถามmx

Comprehensive - Obstetrics & Gynecology                                          หนาที่ 36              Comprehensive - Obstetrics & Gynecology                                              หนาที่ 37
Compre si 2010 l
Compre si 2010 l
Compre si 2010 l
Compre si 2010 l
Compre si 2010 l
Compre si 2010 l
Compre si 2010 l
Compre si 2010 l
Compre si 2010 l
Compre si 2010 l
Compre si 2010 l
Compre si 2010 l
Compre si 2010 l

More Related Content

What's hot

Osce examination si116
Osce examination si116Osce examination si116
Osce examination si116vora kun
 
Mdcu Exam Step 2 2010
Mdcu Exam Step 2 2010Mdcu Exam Step 2 2010
Mdcu Exam Step 2 2010vora kun
 
Step3 Tutorial by SWU book1
Step3 Tutorial by SWU book1Step3 Tutorial by SWU book1
Step3 Tutorial by SWU book1vora kun
 
NTstep3round2 9_jan2554
NTstep3round2 9_jan2554NTstep3round2 9_jan2554
NTstep3round2 9_jan2554vora kun
 
Nt2553step3round1 28NOV2553
Nt2553step3round1 28NOV2553Nt2553step3round1 28NOV2553
Nt2553step3round1 28NOV2553vora kun
 
2007821172158 466 6438_1
2007821172158 466 6438_12007821172158 466 6438_1
2007821172158 466 6438_1New Srsn
 
Osce ศรว ครั้งที่สอง 10jan53
Osce ศรว ครั้งที่สอง 10jan53Osce ศรว ครั้งที่สอง 10jan53
Osce ศรว ครั้งที่สอง 10jan53vora kun
 
ข้อสอบNt จัดพิมพ์โดย med pnu3 goal100
ข้อสอบNt จัดพิมพ์โดย med pnu3 goal100ข้อสอบNt จัดพิมพ์โดย med pnu3 goal100
ข้อสอบNt จัดพิมพ์โดย med pnu3 goal100Hummd Mdhum
 
Nt2009 complete all
Nt2009 complete allNt2009 complete all
Nt2009 complete allvora kun
 
5696 6770-1-pb
5696 6770-1-pb5696 6770-1-pb
5696 6770-1-pbNew Srsn
 
Case study surgery
Case study surgeryCase study surgery
Case study surgerysoftmail
 

What's hot (17)

Osce examination si116
Osce examination si116Osce examination si116
Osce examination si116
 
Example osce
Example osceExample osce
Example osce
 
National test _2553_TU
National test _2553_TUNational test _2553_TU
National test _2553_TU
 
Key word osce
Key word osceKey word osce
Key word osce
 
Mdcu Exam Step 2 2010
Mdcu Exam Step 2 2010Mdcu Exam Step 2 2010
Mdcu Exam Step 2 2010
 
Step3 Tutorial by SWU book1
Step3 Tutorial by SWU book1Step3 Tutorial by SWU book1
Step3 Tutorial by SWU book1
 
NTstep3round2 9_jan2554
NTstep3round2 9_jan2554NTstep3round2 9_jan2554
NTstep3round2 9_jan2554
 
Nt2553step3round1 28NOV2553
Nt2553step3round1 28NOV2553Nt2553step3round1 28NOV2553
Nt2553step3round1 28NOV2553
 
2007821172158 466 6438_1
2007821172158 466 6438_12007821172158 466 6438_1
2007821172158 466 6438_1
 
Osce ศรว ครั้งที่สอง 10jan53
Osce ศรว ครั้งที่สอง 10jan53Osce ศรว ครั้งที่สอง 10jan53
Osce ศรว ครั้งที่สอง 10jan53
 
ข้อสอบNt จัดพิมพ์โดย med pnu3 goal100
ข้อสอบNt จัดพิมพ์โดย med pnu3 goal100ข้อสอบNt จัดพิมพ์โดย med pnu3 goal100
ข้อสอบNt จัดพิมพ์โดย med pnu3 goal100
 
Nt2009 complete all
Nt2009 complete allNt2009 complete all
Nt2009 complete all
 
5696 6770-1-pb
5696 6770-1-pb5696 6770-1-pb
5696 6770-1-pb
 
Case study surgery
Case study surgeryCase study surgery
Case study surgery
 
Case01
Case01Case01
Case01
 
Nl part ii march 2009
Nl part ii march 2009Nl part ii march 2009
Nl part ii march 2009
 
Cpg ped
Cpg pedCpg ped
Cpg ped
 

Similar to Compre si 2010 l

Exercise national license_part_ii_march_2009_2
Exercise national license_part_ii_march_2009_2Exercise national license_part_ii_march_2009_2
Exercise national license_part_ii_march_2009_2Loveis1able Khumpuangdee
 
Nle step 2_2009 si115-116 and nle_step_2_2009 nctms editors cut
Nle step 2_2009 si115-116 and nle_step_2_2009 nctms editors cutNle step 2_2009 si115-116 and nle_step_2_2009 nctms editors cut
Nle step 2_2009 si115-116 and nle_step_2_2009 nctms editors cutLoveis1able Khumpuangdee
 
interesting case
interesting  caseinteresting  case
interesting caseSHAMONBEST1
 
Rupture appendicitis
Rupture appendicitisRupture appendicitis
Rupture appendicitissoftmail
 
แนวทางการดูแลรักษาผู้ป่วยไวรัสตับอักเสบบีและซีเรื้อรัง
แนวทางการดูแลรักษาผู้ป่วยไวรัสตับอักเสบบีและซีเรื้อรังแนวทางการดูแลรักษาผู้ป่วยไวรัสตับอักเสบบีและซีเรื้อรัง
แนวทางการดูแลรักษาผู้ป่วยไวรัสตับอักเสบบีและซีเรื้อรังUtai Sukviwatsirikul
 
Present blood program
Present blood programPresent blood program
Present blood programTHANAKORN
 
Nle step 2_2009 si115-116 and nle_step_2_2009 nctms editors cut key
Nle step 2_2009 si115-116 and nle_step_2_2009 nctms editors cut keyNle step 2_2009 si115-116 and nle_step_2_2009 nctms editors cut key
Nle step 2_2009 si115-116 and nle_step_2_2009 nctms editors cut keyLoveis1able Khumpuangdee
 
Nl 2010 nctms
Nl 2010 nctmsNl 2010 nctms
Nl 2010 nctmsNew Srsn
 
แนวทางการรกัษาไวรัสตับอกัเสบบีในผู้ใหญ่ รศ.นพ.พิศาล ไม้เรียง
แนวทางการรกัษาไวรัสตับอกัเสบบีในผู้ใหญ่ รศ.นพ.พิศาล ไม้เรียง แนวทางการรกัษาไวรัสตับอกัเสบบีในผู้ใหญ่ รศ.นพ.พิศาล ไม้เรียง
แนวทางการรกัษาไวรัสตับอกัเสบบีในผู้ใหญ่ รศ.นพ.พิศาล ไม้เรียง Utai Sukviwatsirikul
 
tumor conferenddddddddddddddce 24-11-66.pptx
tumor conferenddddddddddddddce 24-11-66.pptxtumor conferenddddddddddddddce 24-11-66.pptx
tumor conferenddddddddddddddce 24-11-66.pptxChanyutTuranon1
 
24.hbv and hcv guideline 2012 (update)
24.hbv and hcv guideline 2012 (update)24.hbv and hcv guideline 2012 (update)
24.hbv and hcv guideline 2012 (update)Aimmary
 
Centre management - PD quality นพ.สกานต์
Centre management -  PD quality นพ.สกานต์Centre management -  PD quality นพ.สกานต์
Centre management - PD quality นพ.สกานต์Kamol Khositrangsikun
 

Similar to Compre si 2010 l (20)

Compre step 2_2010 si key
Compre step 2_2010 si keyCompre step 2_2010 si key
Compre step 2_2010 si key
 
Exercise national license_part_ii_march_2009_2
Exercise national license_part_ii_march_2009_2Exercise national license_part_ii_march_2009_2
Exercise national license_part_ii_march_2009_2
 
Nl part ii march 2009
Nl part ii march 2009Nl part ii march 2009
Nl part ii march 2009
 
Nle step 2_2009 si115-116 and nle_step_2_2009 nctms editors cut
Nle step 2_2009 si115-116 and nle_step_2_2009 nctms editors cutNle step 2_2009 si115-116 and nle_step_2_2009 nctms editors cut
Nle step 2_2009 si115-116 and nle_step_2_2009 nctms editors cut
 
interesting case
interesting  caseinteresting  case
interesting case
 
Rupture appendicitis
Rupture appendicitisRupture appendicitis
Rupture appendicitis
 
National license 2010 by med tu 16
National license 2010 by med tu 16National license 2010 by med tu 16
National license 2010 by med tu 16
 
แนวทางการดูแลรักษาผู้ป่วยไวรัสตับอักเสบบีและซีเรื้อรัง
แนวทางการดูแลรักษาผู้ป่วยไวรัสตับอักเสบบีและซีเรื้อรังแนวทางการดูแลรักษาผู้ป่วยไวรัสตับอักเสบบีและซีเรื้อรัง
แนวทางการดูแลรักษาผู้ป่วยไวรัสตับอักเสบบีและซีเรื้อรัง
 
Bleeding in first half
Bleeding in first halfBleeding in first half
Bleeding in first half
 
Cpg anemia
Cpg anemiaCpg anemia
Cpg anemia
 
Present blood program
Present blood programPresent blood program
Present blood program
 
Hiv รพช2015 2.ppt
Hiv รพช2015  2.pptHiv รพช2015  2.ppt
Hiv รพช2015 2.ppt
 
Nle step 2_2009 si115-116 and nle_step_2_2009 nctms editors cut key
Nle step 2_2009 si115-116 and nle_step_2_2009 nctms editors cut keyNle step 2_2009 si115-116 and nle_step_2_2009 nctms editors cut key
Nle step 2_2009 si115-116 and nle_step_2_2009 nctms editors cut key
 
Interesting case
Interesting caseInteresting case
Interesting case
 
Nl 2010 nctms
Nl 2010 nctmsNl 2010 nctms
Nl 2010 nctms
 
แนวทางการรกัษาไวรัสตับอกัเสบบีในผู้ใหญ่ รศ.นพ.พิศาล ไม้เรียง
แนวทางการรกัษาไวรัสตับอกัเสบบีในผู้ใหญ่ รศ.นพ.พิศาล ไม้เรียง แนวทางการรกัษาไวรัสตับอกัเสบบีในผู้ใหญ่ รศ.นพ.พิศาล ไม้เรียง
แนวทางการรกัษาไวรัสตับอกัเสบบีในผู้ใหญ่ รศ.นพ.พิศาล ไม้เรียง
 
tumor conferenddddddddddddddce 24-11-66.pptx
tumor conferenddddddddddddddce 24-11-66.pptxtumor conferenddddddddddddddce 24-11-66.pptx
tumor conferenddddddddddddddce 24-11-66.pptx
 
24.hbv and hcv guideline 2012 (update)
24.hbv and hcv guideline 2012 (update)24.hbv and hcv guideline 2012 (update)
24.hbv and hcv guideline 2012 (update)
 
Rr rx
Rr rxRr rx
Rr rx
 
Centre management - PD quality นพ.สกานต์
Centre management -  PD quality นพ.สกานต์Centre management -  PD quality นพ.สกานต์
Centre management - PD quality นพ.สกานต์
 

More from vora kun

ประชุมวิชาการ ศิริราช 53
ประชุมวิชาการ ศิริราช 53ประชุมวิชาการ ศิริราช 53
ประชุมวิชาการ ศิริราช 53vora kun
 
ประชุมวิชาการ ศิริราช 52
ประชุมวิชาการ ศิริราช 52ประชุมวิชาการ ศิริราช 52
ประชุมวิชาการ ศิริราช 52vora kun
 
CPR 2010 อ ปริญญา รามา
CPR 2010 อ ปริญญา รามาCPR 2010 อ ปริญญา รามา
CPR 2010 อ ปริญญา รามาvora kun
 
Osce คณะ si 115
Osce คณะ si 115Osce คณะ si 115
Osce คณะ si 115vora kun
 
NT step2 march 53
NT step2 march 53NT step2 march 53
NT step2 march 53vora kun
 
Thai Osteoporosis guideline 2553
Thai Osteoporosis guideline 2553Thai Osteoporosis guideline 2553
Thai Osteoporosis guideline 2553vora kun
 
Abnormal pap smear ศิริราช ppt
Abnormal pap smear ศิริราช pptAbnormal pap smear ศิริราช ppt
Abnormal pap smear ศิริราช pptvora kun
 
ortho 02 orthopaedic complication & prevention + orthopaedic trauma (practica...
ortho 02 orthopaedic complication & prevention + orthopaedic trauma (practica...ortho 02 orthopaedic complication & prevention + orthopaedic trauma (practica...
ortho 02 orthopaedic complication & prevention + orthopaedic trauma (practica...vora kun
 
ortho 06 common ortho dis 2 edited 12 mar 10
ortho 06 common ortho dis 2 edited 12 mar 10ortho 06 common ortho dis 2 edited 12 mar 10
ortho 06 common ortho dis 2 edited 12 mar 10vora kun
 
ortho 05 common rheumatic dx rx
ortho 05 common rheumatic dx rxortho 05 common rheumatic dx rx
ortho 05 common rheumatic dx rxvora kun
 
ortho 01 management of open fracture-update by kk 31052010
ortho 01 management of open fracture-update by kk 31052010ortho 01 management of open fracture-update by kk 31052010
ortho 01 management of open fracture-update by kk 31052010vora kun
 
ortho 04 drugs in orthopaedic (principle & common use)
ortho 04 drugs in orthopaedic (principle & common use)ortho 04 drugs in orthopaedic (principle & common use)
ortho 04 drugs in orthopaedic (principle & common use)vora kun
 
ortho 03 principle of closed reduction in fracture and dislocation
ortho 03 principle of closed reduction in fracture and dislocationortho 03 principle of closed reduction in fracture and dislocation
ortho 03 principle of closed reduction in fracture and dislocationvora kun
 
ortho 02 orthopaedic complication & prevention + orthopaedic trauma (practica...
ortho 02 orthopaedic complication & prevention + orthopaedic trauma (practica...ortho 02 orthopaedic complication & prevention + orthopaedic trauma (practica...
ortho 02 orthopaedic complication & prevention + orthopaedic trauma (practica...vora kun
 
หัตถการที่จำเป็นทางสูติ
หัตถการที่จำเป็นทางสูติหัตถการที่จำเป็นทางสูติ
หัตถการที่จำเป็นทางสูติvora kun
 
SWU CXR interpretation
SWU  CXR interpretationSWU  CXR interpretation
SWU CXR interpretationvora kun
 
Step3 Tutorial by SWU book2
Step3 Tutorial by SWU book2Step3 Tutorial by SWU book2
Step3 Tutorial by SWU book2vora kun
 
Total parenteral nutrition
Total parenteral nutritionTotal parenteral nutrition
Total parenteral nutritionvora kun
 
NeuroSx step2 Review
NeuroSx step2 ReviewNeuroSx step2 Review
NeuroSx step2 Reviewvora kun
 
ศรว 51 By Cmu
ศรว 51 By Cmuศรว 51 By Cmu
ศรว 51 By Cmuvora kun
 

More from vora kun (20)

ประชุมวิชาการ ศิริราช 53
ประชุมวิชาการ ศิริราช 53ประชุมวิชาการ ศิริราช 53
ประชุมวิชาการ ศิริราช 53
 
ประชุมวิชาการ ศิริราช 52
ประชุมวิชาการ ศิริราช 52ประชุมวิชาการ ศิริราช 52
ประชุมวิชาการ ศิริราช 52
 
CPR 2010 อ ปริญญา รามา
CPR 2010 อ ปริญญา รามาCPR 2010 อ ปริญญา รามา
CPR 2010 อ ปริญญา รามา
 
Osce คณะ si 115
Osce คณะ si 115Osce คณะ si 115
Osce คณะ si 115
 
NT step2 march 53
NT step2 march 53NT step2 march 53
NT step2 march 53
 
Thai Osteoporosis guideline 2553
Thai Osteoporosis guideline 2553Thai Osteoporosis guideline 2553
Thai Osteoporosis guideline 2553
 
Abnormal pap smear ศิริราช ppt
Abnormal pap smear ศิริราช pptAbnormal pap smear ศิริราช ppt
Abnormal pap smear ศิริราช ppt
 
ortho 02 orthopaedic complication & prevention + orthopaedic trauma (practica...
ortho 02 orthopaedic complication & prevention + orthopaedic trauma (practica...ortho 02 orthopaedic complication & prevention + orthopaedic trauma (practica...
ortho 02 orthopaedic complication & prevention + orthopaedic trauma (practica...
 
ortho 06 common ortho dis 2 edited 12 mar 10
ortho 06 common ortho dis 2 edited 12 mar 10ortho 06 common ortho dis 2 edited 12 mar 10
ortho 06 common ortho dis 2 edited 12 mar 10
 
ortho 05 common rheumatic dx rx
ortho 05 common rheumatic dx rxortho 05 common rheumatic dx rx
ortho 05 common rheumatic dx rx
 
ortho 01 management of open fracture-update by kk 31052010
ortho 01 management of open fracture-update by kk 31052010ortho 01 management of open fracture-update by kk 31052010
ortho 01 management of open fracture-update by kk 31052010
 
ortho 04 drugs in orthopaedic (principle & common use)
ortho 04 drugs in orthopaedic (principle & common use)ortho 04 drugs in orthopaedic (principle & common use)
ortho 04 drugs in orthopaedic (principle & common use)
 
ortho 03 principle of closed reduction in fracture and dislocation
ortho 03 principle of closed reduction in fracture and dislocationortho 03 principle of closed reduction in fracture and dislocation
ortho 03 principle of closed reduction in fracture and dislocation
 
ortho 02 orthopaedic complication & prevention + orthopaedic trauma (practica...
ortho 02 orthopaedic complication & prevention + orthopaedic trauma (practica...ortho 02 orthopaedic complication & prevention + orthopaedic trauma (practica...
ortho 02 orthopaedic complication & prevention + orthopaedic trauma (practica...
 
หัตถการที่จำเป็นทางสูติ
หัตถการที่จำเป็นทางสูติหัตถการที่จำเป็นทางสูติ
หัตถการที่จำเป็นทางสูติ
 
SWU CXR interpretation
SWU  CXR interpretationSWU  CXR interpretation
SWU CXR interpretation
 
Step3 Tutorial by SWU book2
Step3 Tutorial by SWU book2Step3 Tutorial by SWU book2
Step3 Tutorial by SWU book2
 
Total parenteral nutrition
Total parenteral nutritionTotal parenteral nutrition
Total parenteral nutrition
 
NeuroSx step2 Review
NeuroSx step2 ReviewNeuroSx step2 Review
NeuroSx step2 Review
 
ศรว 51 By Cmu
ศรว 51 By Cmuศรว 51 By Cmu
ศรว 51 By Cmu
 

Compre si 2010 l

  • 1. Comprehensive ป 5 วันที่ 13 กุมภาพันธ 2553 ณ โรงพยาบาลศิริราช คณะแพทยศาสตรศิริราชพยาบาล ขอสอบแบงออกเปน รอบเชา 150 ขอ 8.30 – 12.00 น. และรอบบาย 150 ขอ 13.00 – 16.30 น. ลักลอบออกมาอยางถูกตอง - - BY SI116 - - ขอบคุณเพื่อนๆ SI116 ทุกคนที่ชวยกันจําออกมานะ ขอบคุณเพื่อนๆ ที่ชวยกันพิมพ รวบรวม และเรียบเรียง พี่สาระ194 กานต008 กานต011 อรณ015 จอย035 ออบ043 ปง056 ชา058 โอม69 ธัญ079 นอร082 แพร085 แนท089 ปาน090 แปก097 ปอ108 พลอย118 แบงค144 ถิงถิง147 นุย157 เจ160 หยก161 มิ้ม173 บอย180 ตุกตา183 โย191 ไปฟ201 ญา217 แน็ต221 เบลล222 พลอย226 ปลาม230 บิ๋ว242 กาวหนาPI* และคนอื่นๆถาลืมไปตองขออภัยดวยนะ ..สุดทาย..ปกนารักๆ โดย กิ๊บ106 แถมๆๆ สถิติของการสอบจากผูเขาสอบจํานวน 348 คน Max 74.92%, Min 35.12%, Mean 59.87%, Reliability 0.82, Standard error of measurement (SEM) 7.38% เกณฑการตัดสินผลการสอบ: เกณฑขั้นต่ํา 60% นําคา SEM มาพิจารณาคะแนนต่ําสุดที่ สามารถสอบผานโดยใชระดับความเชื่อมั่น 99% ดังนั้น คะแนนต่ําสุดที่สอบผาน 53.63 %
  • 2. - - - - - - - - - - - - - - - - 01 Medicine - - - - - - - - - - - - - - - - Contents <<<<< Hematology >>>>> 1. ผูปวยหญิงอายุ 25 ป มาพบแพทยดวยอาการประจําเดือนมามาก ครั้งละ 10 วัน มา 2 01 Medicine P. 1 เดือน มีเลือดออกตามไรฟนมา 2 สัปดาห ตรวจรางกายพบซีดเล็กนอย ไมเหลือง ตับ- มาม-ตอมน้ําเหลืองไมโต มีจุดเลือดออกใตผิวหนังตามแขนขา CBC = Hb 10 , Hct 30% 02 Pediatrics P. 20 WBC 6000 , N 75% , L20% ,MCV 71 fl, WBC 6000, Plt 10000 รักษา ??? A. azathriopine B. dexamethazone 03 Obstetrics and Gynecology P. 36 C. vincristine D. platelet transfusion E. IVIG 04 Surgery P. 40 2. ลูกคนแรกเปนHbH ตรวจพอ Hb13 MCV65 reticulocyte count1% HbA92% HbA25.5% แม Hb11 MCV60 reticulocyte count0.5% HbA91% HbE19% ลูกคนตอไปจะ 05 Orthopaedics and rehab P. 43 มีโอกาสไมเปนโรคและพาหะของโรคกี่เปอรเซ็นต A. 0% B. 12.5% 06 Eye and ENT P. 47 C. 25% D. 37.5% E. 50% 07 Psychiatry P. 51 3. ผูปวยชายอายุ 70 ป มีตอมน้ําเหลืองที่ คอโต 2 ขาง มีซีด เหลืองเล็กนอย ตรวจรางกาย ปกติ ตับปกติ ยกเวนมามโต 08 Forensic Med and Ethics P. 55 A. lymphoma with bone metastatis B. diffused TB 09 Miscellaneous P. 60 C. CLL D. CML with extramedullary blastic formation Comprehensive - Medicine หนาที่ 1
  • 3. 4. มารดาคลอดบุตร ตายตั้งแตแรกคลอด ทารกบวม แม Hb 11 MCV 60 พอ Hb 12.5 C. LDH D. ดู blood smear MCV 55 ขอใดนาจะเปน Hb typing ของพอ 9. หญิง มีซีด เหลือง plt. ต่ํา จะทําอะไร A. - -/α α B. - α/α α c.direct coomb test C. - -/ - α D. - -/- α cs E. - -/- - <<<<< GI >>>>> 5. ผูปวยมาดวย RUQ pain & moderate jaundice มา 2 วัน ตรวจ U/S พบ common bile 10. ผูปวยหญิง ถายอุจจาระเละ 3 ครั้ง/วัน มา 2 ป มักเปนเวลากินเผ็ด นม ครีม ปวดทอง มี duct stone ไดใหการรักษาดวย ceftriaxone มา 5 วัน อาการไมดีขึ้น ลมมากในทอง ผายลม ครั้งนี้ตรวจพบ PT prolong , normal aPTT ควรใหการรักษาเบื้องตนอยางไร A. Lactose intolerance B. IBS C. CA colon A. vitamin K B. FFP 11. ผูปวยหญิงอายุ 30 ป ปวดทองนอยขางขวามา 4 ชั่วโมง ปวดมากทันที ไมราวไปที่ใด C. observe N/V 1-2 ครั้ง มีถายเหลว 1 ครั้ง ไมมีปสสาวะแสบขัด ไมมีตกขาวผิดปกติ ไมมีไข LMP 1 6. ชายแก มาดวยออนเพลีย ไข coomb + เดือนกอน ตรวจรางกาย T 37oC , not pale , no jaundice , abdomen : flat, mild tenderness A. G-6 PD at RLQ , CVA not tenderness , CBC : Hct. 37% , WBC 9,000 cell/mm3 , Plt. 250,000 B. CLL with AIHA /mm3 UA : Sp.Gr. 1.020 , WBC 2-3 /HP , RBC 5-10 /HP จงวินิจฉัยโรค C. Acute leukemia A. acute gastroenteritis B. acute appendicitis 7. ผูหญิงกินมังสวิรัติ เครงมาก (ไมกินหมู,ไข) กินเตาหู ถั่ว ธัญพืช กินผักใบเขียว ผูปวย C. acute pyeronephritis D. ureteric stone เสี่ยงที่จะเกิดภาวะใด E. ectopic pregnancy A. folate def. B. B12 def. 12. ช 50 yr ดื่มเหลาขาว ½ ขวด /วัน มา 10 ป ทองบวม ขาโต ตัวเหลือง ตาเหลือง C. Fe def. D. ขาด Ca abdomen soft not tender liver not palpable spleen 2 FB shifting dullness E. ขาด protein +fluid thrill + LFT TB DB สูง SGOT SGPT ปกติ albumin ปกติ ALP สูง Glo 4+ 8. ผูปวยหญิงอายุ 50 ป มี petechiae ที่ขาทั้ง 2 ขาง + มีเลือดออกตามไรฟน PE: ปกติ ; Manage ที่ไมเหมาะสมคือ CBC : RBC ปกติ , WBC ปกติ(N70%, L15%, E or Mวะ?? 15%), Plt. 15000 ตรวจอะไร A. ERCP B. Ultrasound liver เพิ่มเติมเพื่อชวยวินิจฉัย C. EGD D. Abdominal paracentsis A. bleeding time B. coagulogram 13. แสบรอนแนนหนาอกมา 1 m, ECG: WNL, มักเปนหลังอาหาร กอนนอน Mx Comprehensive - Medicine หนาที่ 2 Comprehensive - Medicine หนาที่ 3
  • 4. A. simethicone B. omeprazole A. urine B. ANA C. antacid D. oral nitroglycerine C. RF 14. hepatitis C มีไข , ascites , ขาบวม SAAG มากกวาเทากับ 1.1 จะรักษาอยางไร 18. หญิง30ป ปวดขอนิ้วมือและขอมือ2ขาง 4 เดือน ตรวจพบ symmetrical arthritis of both A. ให albumin wrists MCP, PIP joint both hand และพบ joint space narrowing & ulnar styloid erosion B. ceftriaxone 15 d + somatostatin IV จงใหการวินิจฉัย C. cipoflox oral + teripressin IV A. Osteoarthritis B. Reumatoid arthritis D. ceftriaxone 29 d C. Reactive arthritis D. SLE 15. ผูชายอายุ 50 ป มีอาการ cirrhosis pitting edema 2+ , jaundice , spider nevi สงตรวจ E. .scleroderma อะไรเพื่มเติม 19. หญิงอายุ25ป มีอาการปวดขอนิ้ว ขอมือ ศอก เขาทั้งสองขาง มาประมาณ4เดือน ขอติด A. HBS Ag , Anti HCV , Anti HBC ตอนเชา~2ชม. มีอาการออนเพลีย น้ําหนักลด4kg ใน 4เดือน ตรวจรางกาย T37.4 BPและ B. HBS Ag , HCV Ag , Anti HAV P ปกติ พบinflammation ของ 2nd, 3rd Rt. MCP 16. ผูปวยหญิง อายุ 46 ป ไมมีอาการผิดปกติใดๆ ตรวจรางกายปกติ ดื่มสุราเปนครั้งคราว 3rd, 4th Lt. PIP, both wrist, elbow, knee จงใหการวินิจฉัย ดื่มแตเบียร มากพอสมควร BMI 26 , waist circum 92 , BP normal , FBS impaired , AST A. SLE B. Rheumatoid arthritis elevated , ALT elevated (AST<ALT) ดื่ม alcohol ครั้งคราว มา 5 ป ขอใดคือสาเหตุสําคัญ C. Rheumatic fever D. Undiff spondyloarthrepathy ที่ทําให liver enzyme ขึ้นในผูปวยรายนี้ E. Ankylosing spondylitis A. chronic viral hepatitis B. alcoholic fatty liver disease 20. ชายปวด right ankle BMI สูง , FBS 123 , LDL 100 , HDL 50 ให colchicin แลวอาการ C. autoimmune hepatitis D. non – alcoholic steatosis ดีขึ้น ทําอยางไรตอ E. non – alcoholic steatohepatitis A. exercise B. allopurinol C. gemfibrocil D. simvastatin <<<<< Rheumatology >>>>> 17. หญิงอายุ 25 ป มีอาการปวดขอมือ ปวดขอนิ้วมา 1 สัปดาหกอน มีผื่นผิวหนัง malar <<<<< Nephrology >>>>> rash, oral ulcer U/A RBC 10-20/HF, few granular casts, proteinuria จะสง Investigation อะไรตอ Comprehensive - Medicine หนาที่ 4 Comprehensive - Medicine หนาที่ 5
  • 5. 21. ผูหญิง 30 ป ปสสาวะเปนสีน้ําลางเนื้อ 3 วันกอน มีไข ไอ เจ็บคอ ซื้อยามากินเอง PE C. Vit B12 1.2 mcg/day D. Cholinesterase inhibitor T= 38 no edema, BP 150/70 mmHg, mild pharynx injection, lung clear, cvs normal U/A E. Benzodiazepine RBC 100/HPF, WBC 5-10/HPF จงวินิจฉัย 25. ชาย 25 ป โรงงานเทียน มีสวนประกอบethanol สารเรงการแข็งตัวของเทียน ทะเลาะ A. IgA nephropathy กับหัวหนา กินอะไรเขาไป แลวหมดสติ คิดวาเกิดจากอะไร B. PSGN A. Alcohol intoxication B. Alcoholic psychosis C. Ureteric stone C. Hypoglycemia D. Hyporkalemia metabolic acidosis D. Acute pyelonephritis E. Acute interstitial nephritis <<<<< Endocrinology&Metabolic >>>>> 22. ชาย 60 ปมาดวยปวดเอวซาย fever with chill ปสสาวะสีเขม 26. ชายอายุ 50 ป มีอาการมึนศีรษะ หนักทายทอยมา 1 เดือน ตรวจรางกาย BP 170/70 PE: Lt CVA tenderness, UA : WBC 50-100/HP , RBC 5-10/HF mmHg ระบบอื่นๆปกติ ผูปวยเปนความดันโลหิตสูงมา 1 ป ไมไดรักษาใดๆ FBS 80 CBC : leukocytosis ( N=80, L=20 ) mg/dL BUN 20 Cr 1.0 electrolyte อื่นๆปกติ ควรใหรักษาผูปวยอยางไร จะสงตรวจทางหองปฏิบัติการที่เหมาะสมตอไปอยางไร A. HCTZ 25 mg OD B. Enalapril 10 mg bid A. U/S KUB B. IVP C. Nifedipine 5 mg tid D. Lorsartan 100 mg OD C. Retrogradeurethrography E. Propanolol 10 mg tid 23. On NSAID (no cast) 27. ผูปวยชายอายุ 47ป blood sugar 250 , HDL 30 , Cholesterol 193 , LDL 130 , TG 180 A. AIN B. acute tubular necrosis ผูปวยมี rish ตอ artherosclerosis ระดับใด และ goal ในการควบคุมเปนอยางไร A. very high risk , LDL<70 <<<<< Toxicology >>>>> B. very high risk , LDL<70 , HDL>40 24. ผูปวยชายอายุ 45 ป มีประวัติ heavy alcohol drinking เคยมีอาการสับสนเมื่อขณะลด C. very high risk , LDL<70 , HDL>40 , TG<150 ปริมาณการดื่มเมื่อ 6 เดือนกอน หลังจากนั้นมีอาการหลงลืม อารมณเฉยเมย พูดในสิ่งที่ D. high risk , LDL<100 ไมเคยเกิดขึ้น รักษา E. high risk , LDL < 100 , HDL > 40 A. Vit E 1,000 IU/day B. Vit B1 100 mg/day Comprehensive - Medicine หนาที่ 6 Comprehensive - Medicine หนาที่ 7
  • 6. 28. ชายอายุ 35 ป มีอาการเหนื่อยงาย ใจสั่น ตรวจรางกายพบวามี thyroid nodule ตรวจทาง 33. หญิง14 ป มีกอนที่คอเห็นไดชัด ไมเจ็บ ไมมีใจสั่น ไมมีน้ําหนักลด PE : generalized หองปฏิบัติการ TFT พบ T3, T4 สูง TSH ต่ํา จะสงตรวจเพิ่มเติมอะไรตอ enlargement of thyroid gland not tender , TFT euthyroid Mx ที่เหมาะสม A. Thyroid scan B. Thyroid antibody A. thyroid scan B. FNA C. CT D. MRI C. Anti-microsomal Anti thyroglobulin E. FNA D.ให thyroid supplement ไปกอน 29. thyroid nodule : T4 ต่ํา TSH สูง จะ confirm ดวยอะไรกอน E. subtotal thyroidectomy A. thyroid scan B. FNA 34. แฟนพาผูปวยหญิงมาสงรพ. เปน DM on mixtard 70/30 40 U แบงใหเชา เย็น ac C. biopsy วันนี้ไมไดกินขาวเย็นแตฉีด mixtard เทาเดิม ตรวจรางกาย PR 100 RR 18 ซึม NS : 30. ผูปวยหญิงอายุ 30 ป มีอาการเหนื่อยงาย ใจสั่น ประมาณ hyperthyroid T4 สูง T3 สูง response to pain , CBG 26 mg/dl , จง management ที่เหมาะสม TSH ลด, thyroid gland: diffuse enlargement A. NSS 1,000 ml/hr A. Subacute thyroiditis B. Grave’s disease B. RI 0.1 U/kg IV bolus + RI 0.1 U/kg/hr C. Hashimoto thyroiditis C. 5%D/N/2 80 ml/hr 31. ชาย 25 ป หิวน้ํา ฉี่บอย 3 สัปดาห มี deep breathing pattern มี BS 300, Na 130, K 2.5, D. 50% glucose 50 ml IV push Ketone 8, BUN 10 initial Mx คือ? 35. ชายสุขภาพดี FBS 400 ควรแนะนํา A. NSS B. RI A. exercise C. K after adequate renal function B.ใหยาเบาหวาน D. K2PO4 after adequeate renal function 36. หญิงวัยกลางคน อาการ clinical Hyperthyroid ใจสั่นเหนื่อยงาย thyroid diffuse 32. ผูปวยหญิงอายุ 60 ป U/D DM type II BW 80 kg, Ht 160 cm ตรวจ FBS ได 180 mg/dl enlarge with thyroid bruit Lab T4 สูง T3สูง TSHต่ํา จะใหการรักษาอยางไร การรักษาเบื้องตน A. metformin B. glinide A. PTU B. B blocker C. sulfonylureas D. alpha-glucosidase inhibitor C. Methimazole D. Lugol solution E. glutadienediane <<<<< Dermatology >>>>> Comprehensive - Medicine หนาที่ 8 Comprehensive - Medicine หนาที่ 9
  • 7. 37. ผูหญิงอายุ 60 ป มีผื่นคันที่ขาหนีบทั้ง 2 ขาง ทายาแลวไมดีขึ้น ผื่นลาย ตรวจรางกายพบ E. bullous impetigo bilateral well defined erythematous patches surrounded with erythematous papule จงให 41. หญิงมีไข มีผื่นที่หนา ตอมาลามไปแขน PE: multiple erythematous papules การรักษา A. Rubella A. Mild potent topical steroid B. Povidine iodine หญิง 18 ป มีตุมขึ้นที่หนา เปนๆ หายๆ 1 ป ไมคัน PE: open and closed comedones at C. Calcineurin inhibitor D. Ketoconazole forehead and cheeks จงใหการรักษา 38. Group of vesicles on erythematous base ที่รมฝปากบนซายสงตรวจ ิ A. 0.25 % tretinoic acid B. 1% clindamycin solution A. KOH preparation C. 4% erythromycin gel D. Oral tretinoin B. Fresh preparation 42. หญิงสามสิบป มีไขต่ํา ๆ มาสามสัปดาห น้ําหนักลดลง หา กิโลกรัมในหนึ่งถึงสอง C. Wright stain เดือน มีผื่นแดงกดเจ็บขึ้นที่หนาแขง PE: multiple discrete erythematous non ulcerative D. Indian ink preparation papule at both shins จงสง Ix for Dx 39. ผูปวยชายอายุ 60 ป มีตุมแดงเจ็บมานาน 1 เดือน PE: multiple tender erythematous A.CBC B.U/A nodules at trunk, arms, legs, multiple infiltrative plaques with ill-defined border at ear C.CXR D.stool exam lobes,back, non caseating granuloma IX สง slit scrape ที่ ear lobe, nasal พบ AF bacilli E.?? (ไมมี skin biopsy) ถาม Dx? 43. ชายมีปวดเมื่อยตามตัว มีผื่นใบหนาลําตัว แขนmultiple discrete vesicle pustule A. Tuberculous leprosy hemorrhagic crust ที่ขา B. Lepromatous leprosy with type I reaction A. chicken pox C. Lepromatous leprosy with type II reaction D. Vasculitis <<<<< Neurology >>>>> E. Cutaneous Tuberculous 44. หญิงอายุประมาณ 30 ป ปวดหัวมาประมาณ 3 เดือน เปนมากตอนตื่นนอน ถาปวดมาก 40. หญิง6 yr U/D lymphoma ไขสูง มีผื่นบริเวณลําตัว PE groups of vesicle pustule ตาม จะมีอาการตามัวรวมดวย มี vertigo, N/V รวม ตรวจรางกายระบบอื่นปกติดี no L4 distribution c demarcated erythematous skin adjacent to the vesicle Dx neurological deficit จงใหการตรวจรักษาที่เหมาะสม A. disseminated varicella B. disseminated herpes A. treat migraine C. varicella c cellulites D. herpes zoster c erysipelas B. treat tension headache Comprehensive - Medicine หนาที่ 10 Comprehensive - Medicine หนาที่ 11
  • 8. C. LP <<<<< Critical care >>>>> D. CT 49. ชายอายุ 70 ป เปน DM นาน on insulin 70:30 เชา 40 unit เย็น 20 unit เชานี้กินขาว E. MRI include cervical spine ไมได แตไมไดลดยาฉีด ตอมามีอาการซึมลงจึงมาโรงพยาบาล เจาะ DTX = 23 mg/dl จง 45. หญิง 30 ป มีอาการปวดหัวมากตอนเชา 30 นาที มีเวียนหัว สัมพันธกับกิจกรรมวิ่ง ควร ใหการรักษา สง investigation A. 50% glucose 50 cc. iv stat A. LP B. CT B. Draw blood sample สง confirm blood sugar C. MRI D. Myelogram C. 10% D/W iv drip 46. ชาย อายุ 20 ป เหมอลอย แขนขาขวาเกร็งกระตุก 2-3 นาที กินยา phenytoin 300mg/d D. สง lab……….. มานาน 5 ป ไดรับการวินิจฉัยเปน cysticercosis หยุดยาเองมา 1 เดือน จงใหการรักษา 50. ผูปวยอายุ 70 ป มี CA colon with brain&lung metastasis Dxเมื่อ 3 เดือนกอน ญาติ A. phenytoin 300 mg/d w/o loading dose ยอมรับได ไม CPR วันนี้ผูปวยซึมลง กินไดลดลง BP drop E1V1M1 จะใหการดูแล B. phenytoin 300 mg/d with oral phenytoin loading dose อยางไร C. phenytoin 300 mg/d with iv phenytoin loading dose A. ET tube B. IV 1000 ml rate 80ml/hr D. phenytoin 300 mg/d with diazepam loading dose C. NG tube feeding -3 meals D. ET tube then suction E. phenytoin 300 mg/d with clobazam 10 mg/d E. Admit 47. ชาย 17 ป แขนขาออนแรง เคยเปนมากอนแลวกอนหนานี้ เมื่อ 3-4 WK กอน เมื่อคืน 51. ผูปวย CA colon with brain and lung metastasis ญาติทราบพยากรณโรคดี ผูปวยมี กนทํางานหนัก และกินอิ่มมาก Dx อาการซึมลง เหนื่อย เสมหะมาก กินไมคอยได มาร.พ. จะทําอยางไร A. hypokalemic periodic paralysis A. ใส ET tube + suction 48. ญ 60 ป ปวดศีรษะรุนแรง คลื่นไสอาเจียน แขนขาขวาออนแรง ตรวจได motor gr. 0/V B. admit และใหญาติเฝาตลอดเวลา BP 190/100 mmHg นาจะเปนอะไร C. Oxigen mask with bag A.venous sinus thrombosis B.basal gg. hemorrhage D. salbutamol NB C. Acute subarachnoid hemorrhage D. Acute MCA occlusion 52. ผูปวยชายอายุ 60 ป BP 140/90mmHg มาทํา Exploratory Laparotomy เพื่อรักษา E. frontal lobe hemorrhage rupture appendicitis หลังจากผาตัด ผูปวยมีอาการหนามืด เปนลมเวลาลุก PE: T 38.5 C, BP 90/70 mmHg, PR 110/min, RR 30/min Comprehensive - Medicine หนาที่ 12 Comprehensive - Medicine หนาที่ 13
  • 9. จงใหการรักษาในผูปวยรายนี้ C. โอกาสเกิด rheumatic fever เทาคนปกติ A. ตรวจ Urine spec. gr. เพื่อพิจารณาเลือกใหสารน้ํา 57. คนไข heat stroke แมพามาหาหมอ เนื่องจากเปนลมบอย มักเปนตอนเขาแถวตอนเชา B. ให 5% Dextose in 0.45% NSS 100 ml/hr ที่ รร. โดยเฉพาะวันที่อากาศรอน เปนจากเหตุใดมากที่สุด C. ให 0.9% NSS 200 ml/hr แลวประเมินซ้ํา A. vasovagal B. orthostatic hypotension 53. ผูปวยกินปูทะเล,แมงดาทะเล ตอมา 1ชม. เกิด n/v กลามเนื้อออนแรง หายใจชา หนามืด C. cardiogenic syncope เปนลม 58. Septicshock ถามวาเกิดจากกลไกลใด A. endotracheal intubation B. adrenaline IM a. ลดpreload b. เพิ่มafter load C. bronchodilator nebulizer D. NSS load c. ลด sympathetic tone d. ลด hemoconcentration E. Lavage and activated charcoal e. ลด cardiac contract 54. ผูปวยหยุดหายใจ pumpขึ้น ไดVF ทําไงตอ 59. สงสัย IE ฟงได Pansystolic murmur Grade 3/6 at Apex , Diastolic murmur Grade 2/6 A. defibแลวpumpตอ at Apex นาจะเกิดจาก valvular lesion ใด a. MR b. MR with relative MS <<<< Cardiology >>>> c. AS 55. ผูปวยหญิง มีประวัติ CA breast ไดทํา modified radical mastectomy ได CMT 6 course ตอมามีอาการหอบเหนื่อยมา 2 สัปดาห JVP up to mandible PE. ฟง korokoff ไมชัดที่ 100 <<<<< RS >>>>> mmHg มีๆหายๆ ที่ 80 mmHg จึงฟงไดชัด คิดวาเขากับโรคโดมากที่สุด 60. ผูปวย CA lung with brain metastasis (Stage IV) admit อยูเพื่อให CMT หลังใหยาครบ A. congestive heart failure B. cardiac temponade 1 วัน ผูปวยมีอาการหอบเหนื่อย, O2 sat 90% ตรวจรางกายปกติ, CXR: no significant D. constrictive pericarditis change from previous CXR ควรสง investigation ใดเพื่อใหไดประโยชน 56. หญิงวัยกลางคนมีอาการเหนื่อยมากขึ้น pulse เร็ว พบ HR total irregularity ,loud S1, A. D-dimer B. ABG palpable P2,opening snap +, diastolic rumbing murmur gr II at apex lung clear C. CTA for pulmonary artery A. มีโอกาสเกิด embolic stroke มากกวา normal 15-20 เทา 61. Acute asthmatic attack Moderate distress ได O2 + salbutamol NB B. โอกาสเกิด IE เทาคนปกติ ผูปวยควรไดอะไรเพิ่มเติม Comprehensive - Medicine หนาที่ 14 Comprehensive - Medicine หนาที่ 15
  • 10. A. Terbutaline SC B. Ipratropium NB C. pulmonary embolism with pulmonary infarction C. Theophyline IV D. Dexamethasone IV D. pulmonary TB E. Beclomethasone NB E. lung cancer 62. ชาย 65 ป HT หอบเหนื่อย แพทยสง CXR สงสัย Lt pleural effusion ควรสงทาไหน A. lt lateral decubitus B. rt lateral decubitus <<<<< Infectious >>>>> C. lordortic view 66. ผูปวยชายอาชีพกรรมกร อายุ 30 ป มีไขสูงมา 4 วัน คลื่นไสอาเจียนไมถายเหลว และ 63. ผูปวยใสมี hydropneumothorax ใส ICD ไดเลือด 400 ml ตอมา dyspnea and ไมมีอาการอยางอื่น ยกเวน คัดจมูก น้ํามูกไหล ไอ Temp 39 C , pharynx injected , tonsil crepitation มี pink frothy sputum Dx enlarged , no jaundice , Liver just palpable mild tenderness จะสงตรวจอะไรเพื่อการ A.reperfusion pulmonary edema B.pneumonia วินิจฉัย C.trauma from ICD D.Atelectasis A. CBC B. H/C 64. ผูปวยชาย 20 ป ทํางานโรงงานพนสีรถยนต มีอาการเหนื่อยหอบ หายใจลําบาก มา C. LFT D. Ab for dengue virus infection นาน 2 ป ตรวจการทํางานของปอดเปน restrictive lung disease ทานจะใหคําแนะนําแก E. Ab for Rickettsia infection ผูปวยรายนี้อยางไร 67. ชายอายุ 25 ป ถายอุจจาระเหลว 2 วัน วันละ 10 ครั้ง ไมปวดทอง อาเจียนวันละ 2-3 A. ยายงานหรือเปลี่ยนตําแหนง ครั้ง PE: T37c, P 110/min, BP 90/60mmHg Ix: CBC Hct 48%, Plt 120,000/mm3, WBC B. ใชอุปกรณปองกันสวนบุคคลขณะทําการพนสี 13,000/mm3 (N 80%, L 20%), BUN 60, Cr 2, Na 142, K 2.8, HCO3 14, Cl 90 ถาม Dx? C. เปลี่ยนเปนน้ํายาพนสีใหม A. Vibrio B. Amoebiasis D. จัดหาพัดลมระบายอากาศ C. Capillaria D. Salmonella 65. ชายอายุ 60 ป U/D severe COPD , 3 วันกอน มีไข เสมหะสีเหลือง PE: V/S temp 38.2 E. isospora C , RR 28 , BP 140/90 , O2Sat 80% , ตรวจ Lung ได fine crepitation RLL ตรวจ chest X- 68. หญิง 35 ป มาดวยมีไข ปวดหัว 10 วันกอนมารพ. T 38 , PR 100/min, BP 100/60, ray พบ alveolar infiltration at RLL , generalized wheezing both lung จงใหการวินิฉัยที่ conjunctival injection, liver just palpable, spleen ไมโต , generalized MP rash at trunk , นาจะเปนไปไดมากที่สุด extremities CBC : Hct 42 % , WBC 7,000 MCV 60 , L 40%, Plt 80,000 diagnosis, total A. COPD exacerbation biliruin 15 B. RLL pneumonia A. Dengue fever B.Scrub typhus Comprehensive - Medicine หนาที่ 16 Comprehensive - Medicine หนาที่ 17
  • 11. C. Leptospirosis D. Typhoid fever E. เชื้อที่ติดเปนเชื้อรุนแรง E. Viral hepatitis 73. คนไขมี dengue fever มีอาการของ rhonchi คิดวาเกิดจากอะไร 69. หญิง 60 ป U/D DM มีผื่นแดงที่หลังคอ มีตุมหนองหลายตุมที่บริเวณผื่นแดง Dx? A. plasma leakage B. Hypovolumic shock A. cabuncle B. cellulitis 74. ผูปวยชายอายุ 23 ป มีผื่นขึ้นมา 1 สัปดาห 3เดือนกอน มีเพศสัมพันธกับเพื่อนสาวที่ C. erysipelas D. ecthyma เพิ่งรูจักกัน อูวว ไมมีแผลที่อวัยวะเพศ PE: ผื่น erythematous บริเวณฝามือฝาเทา และแขน 70. ผูปวย 17 ป มีแขนขาออนแรง 3 วัน เคยมีประวัติ IVDU มากอน ออนแรง lower grade ขาทั้ง 2 ขาง จะรักษาอยางไร 3/5, decrease pain sensation, hyporeflexia, loose sphincter tone, EYE : exudates and A. Ceftriaxone 250mg IM 1 dose hemorrhage at retina นาจะเกิดจากเชื้อใด B. Benzathine Penicillin G 2.4M unit IM 1 dose A. CMV B. EBV C. Benzathine Penicillin G 2.4M unit IM 1dost/week , *3 weeks C. HIV D. S.pneumoniae D. Prednisoline E. S.aureus E. Oral Tetracycline 250mg qid 15 days 71. ชายมีตุมน้ํา(แผลและตุมน้ํา)ที่อวัยวะเพศ เปน ๆ หาย ๆ มา 3-4 ป ชวง 2-3 เดือนเปนถี่ 75. ชายอายุ 30 ป ตรวจพบ Non painful vertical whitish plague along the lateral long ขึ้น ใหการรักษา border of tongue, the lesion is adherent to tongue. รอยโรคดังกลาวนาจะเกิดจากเชื้อใด A. Acyclovir (200) วันละ 5 เม็ก 6 เดือน มากที่สุด B. Acyclovir (400) วันละ 5 เม็ก 6 เดือน A. EBV B. Staphylococcus aureus C. Acyclovir (800) วันละ. 5 ครั้ง 5 วัน C. Streptococcus pneumoniae D. Pox virus D. Acyclovir cream ทาจนกวาจะหาย E. Papilloma virus E. พญายอครีมทา 76. ชาย 30 ป มามโต 1 cm ตับโต 1 cm below costal margin มีไขมา 3 สัปดาห 72. ชายอายุ 25ป เปนโรคลมชัก กินphenytoin tid มา5ป 1วันกอนมีอาการไอ ไข เจ็บคอ ไป A. dengue hemorrhagic fever B. salmonella คลินิกไดATBมากิน ตอมาอาการหนัก ไขขึ้นสูง หนาวสั่น ไปร.พ. แพทยบอกติดเชื้อใน C. murine typhus D. lymphoma กระแสเลือด เกิดจากสาเหตุใด 77. หญิง อายุ 40ป ติดเชื้อCryptococcus แตHIV negative จะตรวจอะไรเพิ่ม A. ATB ไมพอ B. โรคลมชักเสี่ยงpneumoniaจากสําลัก A. IgG B. IgM C. IgA C. ATB ผิดชนิด D. ยาที่ใหทําใหเม็ดเลือดขาวต่ํา D. CD4 E.CD8 Comprehensive - Medicine หนาที่ 18 Comprehensive - Medicine หนาที่ 19
  • 12. - - - - - - -- - - - - - - - - - 02 Pediatrics - - - - - - - - - - - - - - - - <<<<< Neonatology >>>>> 78. เด็กทารกอายุ 12 ชม. มีอาการชักเกร็งกระตุก 2 นาที ระดับน้ําตาลในเลือด 29 mg/dL, น.น.แรกเกิด 2950 g, Ht 47 cm, HC 37 cm ตรวจรางกาย T 37 °C, P 120/min, RR 40/min, BP 78/56 mmHg ตรวจรางกายอื่นๆพบ cleft palate, penis length 2.1 cm, urethral opening at ปลาย penis, testes ทั้ง 2 ขางอยูใน scrotal sac ถาม cause ของ hypoglycemia ในผูปวยรายนี้ที่เปนไปไดมากที่สุดคือ A. Hyperinsulinism B. Gonadotropin deficiency C. Growth hormone deficiency D. Glycogen storage disease E. Primary adrenal insufficiency A. Midgut volvulus B. Intussusception 79. เด็กเพศหญิงคลอดครบกําหนด Hct 50% MB 9.6 mg/dL แรกคลอด น.น. 3000g กวาๆ C. NEC D. Meconium peritonitis ที่ 36 ชั่วโมงหลังคลอด น้ําหนัก 2820 กรัม กินแตนมแมอยางเดียว ศีรษะไม swelling ให 82. เด็กแรกเกิดหนัก 3000g ทํา newborn screening พบวา thyroid function test ที่ day 3 กราฟ CPG jaundice ของเด็ก newborn มา ถามวาจะ manage อยางไร (หรือday5หวา) TSH=?=สูง T4=0.5=ต่ํา ควรทําอยางไร A. ใหเด็กกลับบานได B. ใหเด็กกินนมแมมากขึ้น A. ให Levothyroxine B. ทํา thyroid scan C. phototherapy D. ให IV C. เจาะ TSH & T4 ซ้ํา day14 D. เจาะ T3 o 80. ทารก GA 34 wks BW 1850 g, T 38.5 C, BP ไม drop, abdomen: soft, hypotonia, 83. ทานไดรับมอบหมายใหดูแลทารกคลอดครบกําหนด มีปากแหวงเพดานโหว HC 31 abdominal distention, capillary refill 2 sec หลัง septic W/U ทําอะไรตอ cm มีตาชิดและเล็ก systolic murmur grade 3/6 มือทั้งสองขางมีนิ้วเกิน ทานจะดูแลทารก A. ให empirical ATB B. 0.9% NSS 10 cc/kg V push รายนี้อยางไรกันละหนอ C. Urinary catheter A. บอกพอแมวาไมเปนอันตราย สามารถผาตัดรักษาได 81. ทารกคลอด C/S GA 28 wks due to fetal distress APGAR 5,7 อยู ICU ตลอด 8 วัน มี B. แนะนํากระตุนพัฒนาการ และตรวจการไดยินตอนอายุ 1 ป อาการทองอืด ถายเปนมูกเลือด film abdomen ดังรูป Dx? C. ปรึกษาศัลยตกแตงแกไข ปากแหวงเพดานโหว D. สง CT ดูสมองเพราะหัวเล็ก Comprehensive - Pedriatrics หนาที่ 20 Comprehensive - Pedriatrics หนาที่ 21
  • 13. E. บอกพอแมวาพยากรณโรคในทารกไมดี 88. เด็ก 1 วันกอน ไอ 1 ชั่วโมงกอน ไอมากขึ้น trachea shift to the Rt, mediastinum shift 84. เด็กอายุ 30 สัปดาห มีฟนขึ้น 2 ซี่ลางดานหนา #71, #81 ฟนโยก มารดาพามาพบแพทย to the Rt, decreased breath sounds Rt, CXR ทึบที่ดานขวา ควรสงตรวจอะไรเพิ่มเติมดวย จงใหการวินิจฉัย และ คิดวาเกิดจากความผิดปกติในขั้นตอนใด A. Bronchoscopy B. CXR Rt lateral decubitus A. Neonatal teeth / Initiation phase C. CT B. Natal teeth / Initiation phase C. Neonatal teeth / Proliferative phase <<<<< Infectious >>>>> D. Natal teeth / Proliferative phase E. Supranumeral teeth / Initiation phase 89. เด็กชายอายุ 6 ป ไขต่ําๆ ปวดทอง อาเจียน 2 wks PE เหลืองเล็กนอย ตับโตกดเจ็บ 85. เด็ก preterm GA 34 wks BW1650 g, APGAR 8,9 ที่1,5 min แรกคลอดมีปญหาเรื่อง LAB: SGOT 1500 U/L, SGPT 2500 U/L, anti-HCV neg, anti-HAV IgM pos, anti-HAV การหายใจ feed via OG tube ตั้งแตวันที่ 2 วันนี้อายุ 16 วัน มี apnea 2 times in 30 min, IgG neg, anti-VCA IgG pos, anti-VCA IgM neg, HBs Ag neg, anti-HBc pos, anti-HBs tactile stimulation ได BT 36.3oC, HR186/min, BP 86/56 mmHg, RR 56/min, inactive, pos, mycoplasma titer 1:40 นาจะติดเชื้อใด hypotonia, decreased bowel sounds, distension abdomen หลังจากทํา septic work up แลว A. HAV B. HBV ควรทําอะไรตอเปนอันดับแรก C. HCV D. EBV A. Empirical ATB B. NSS drip 10 cc/kg E. Mycoplasma C. Oxygen hood D. Foley catheterization ติดตาม urine 90. เด็กชายอายุ 6 wks ไอเปนชุดๆจนหนาเขียวที่รอบปากและปลายมือ บางครั้งอาเจียน E. IVIg หลังไอ เวลาที่ไมไอกินนมไดตามปกติ ไมเขียว มีประวัติมารดาไอเปนชุดๆมา 2 wks 86. เด็กคลอดติดไหล หลังคลอดพบแขนซาย flaccid, Lt lung decreased breath sounds and ตรวจรางกาย T 39oC, RR 18/min, no dyspnea, normal breath sounds จงวินิจฉัย movement สาเหตุเกิดจาก A. Acute bronchitis due to RSV A. phrenic nerve injury B. Brachial plexus injury B. Pneumonia due to Chlamydia trachomatis 87. ทารกแรกเกิด แขนขวาไมขยับ ตอนคลอดไหลขวาติด ตองดึงใชแรงมาก ควรตรวจ C. Acute bronchitis due to Mycloplasma pneumoniae อะไรเพิ่มเติม D. Bronchiectasis due to Strep pneumo A. CBC B. Moro reflex E. Pertussis due to Bordetella pertussis C. Asymmetrical tonic neck reflex D. Parachute reflex Comprehensive - Pedriatrics หนาที่ 22 Comprehensive - Pedriatrics หนาที่ 23
  • 14. 91. หญิงอายุ 15 ป ปวดเมื่อย 3 วัน เริ่มมีผื่นขึ้นที่ใบหนา ลําตัว, PE T=39oC, multiple C. Rabie posterior cervical LN ทั้ง 2 ขาง, mild tenderness, mild MP rash at face & trunk, mild 95. เด็กมีไขปวดหัวมา 10 วัน มี injected conjunctivitis, ไมมี eschar, platelet 85000/mcL, pharyngitis, CBC พบ Hct 43%, WBC 9500/mcL (N50%, L50%) plt 150000/mcL จง LFT???, Cr.1.0 จง Dx วินิจฉัย A. Leptospirosis B. Scrub typhus A. Streptococcal pharyngitis B. Viral pharyngitis C. Typhoid fever C. Rubella D. Dengue fever 96. เด็กอายุ 3 ป เจ็บ+ฉี่แสบ PE พบ vulva บวมแดง, swab discharge C/S พบ E. Infectious mononucleosis intracellular Gram negative diplococci จงใหการวินิจฉัย 92. เด็ก (ไมไดบอกอายุ) มีอาการไข 4 วัน เจ็บคอ 5 วัน 2 วันกอนไปพบแพทยไดรับ A. UTI B. Vulvitis + UTI Amoxicillin อาการไมดีขึ้น วันนี้มีผื่นขึ้น PE T 39.0 oC, puffy eyelid, tonsil enlarged, C. Gonorrhea D. UTI+ gonorrhea with child abuse injected with exudate, MP rash at trunk and extremities, Hct 42%, WBC 18000 (N15%, L70%, Atypical Lymph 10%, Monocyte 5%) Plt 18000, พบ Downy cell ใน Peripheral Blood smear <<<<< Nutrition >>>>> จงใหการวินิจฉัย 97. เด็กสงสัย CP แมปอนดวยขาว กระดูกหมู ไข วันละ 3-4 มื้อ มีบวมที่เขาขวา ปวดแดง A. Streptococcal pharyngitis B. Rheumatic heart disease ขอใดถูกตอง C. Infectious mononucleosis A. เลือดออกงายจาก platelet dysfunction 93. เด็กอายุ 4 ป ติดเชื้อบอย ไข ทองเสีย มี oral thrush มี PCP ที่ lung แมไมมี HIV risk จะ B. พบโรคหัวใจเนื่องจาก heart dysfunction สงตรวจอะไร C. กินผักใบเขียวปองกัน A. Ig B. CD4, CD8, CD3, NK D. มีการทํางานผิดปกติของตับ C. Complement D. Phagocyte function 94. เด็กอายุ 6 สัปดาห dyspnea, O2sat. 88%, both upper lung crepitation and rhonchi , <<<<< Development >>>>> ปากเด็กมี oral trush แมไมมีประวัติ HIV คิดถึงอะไร 98. เด็กอายุ 6 ป ตอนนี้เด็กอยูอนุบาล เวลาทําการบานตองใหแมชวย ซนตามวัย เขากับ เด็กชาย 12 ป ชักเกร็ง รูสึกตัว อาปากไมสุด แสยะยิ้ม ถาเสียงดัง แสงจาเปนมากขึ้น Dx เพื่อนได วันนี้แมพามาฉีด vaccine แมกังวลเรื่องลูกจะเรียนไดหรือไม จะประเมินอยางไร A. Tetanus B. Epilepsy A. ใหเด็กเลาเรื่องที่โรงเรียน Comprehensive - Pedriatrics หนาที่ 24 Comprehensive - Pedriatrics หนาที่ 25
  • 15. B. ประเมิน attention span จําของสามอยาง C. Cystitis D. IgA nephropathy C. ใหเด็กวาดรูปคน+รูปเรขาคณิต E. Postinfectious glomerulonephritis D. ขอดูสมุดบันทึกการเรียนจากครู 102. เด็กหญิงอายุ 5 ป มารพ.ดวยทองเสียและอาเจียนมาก ปสสาวะไมออก มา 12 ชม. ได E. ซักประวัติเรื่องเรียนกับแม IV fluid 20 ml/kg ตรวจ serum K 10 mmol/L, Cr 2.5 mg/dL, EKG wide QRS complex 99. เด็กชายอายุ 6 ป พัฒนาการชา พูดเปนคําๆ พอฟงรูเรื่อง เดินไมคอยได ใชชอนกินขาว ขอใดเปนสาเหตุของ EKG ที่ผิดปกตินี้ ไมได ไปกองน้ําเองไมได ถาทําการทดสอบพัฒนาการของเด็กอายุนอยกวา 6 ป จะได IQ A. HyperNa B. HyperK เทาไร C. HyperCa D. HyperMg A. <25 B. 55-70 E. Hyperphosphatemia C. 70-90 D. 90-110 E. >110 <<<<< Allergy >>>>> 103. ผูปวยเด็ก 30 min PTA ถูกผึ้งตอย 15 แผล มีปากบวม ชารอบปาก N/V หายใจหอบ <<<<< Nephrology >>>>> เหนื่อย BP drop, wheezing คิดวาเกิดจากกลไกใด 100. เด็กอายุ 6 ป ปสสาวะสีแดงมา 2 วัน ปสสาวะออกนอยลง ไมเคยเปนมากอน ตรวจ A. Anaphylatic reaction B. Septic shock รางกาย BP 130/90mmHg, puffy eyelid, UA: sp.gr.1.015 protein1+ RBC100-200/HP C. Histamine release syndrome D. Local reaction RBC cast 0-2/LP ไมไดรับการรักษา ปสสาวะออกเองไดดี ยุบบวมใน 4 วัน การตรวจทาง E. Mast cell ปลอยสารออกมา หองปฏิบัติใดนาจะเปนไปไดในผูปวยรายนี้ 104. เด็กหญิงอายุ 6 ป มีผื่นลมพิษทั่วตัว 2 hrs กอนมา รพ. เริ่มมีอาการหลังกินกุง 15 นาที A. Antinuclear Ab titer 1:2560 B. Anti-streptolysin O titer 100 IU/mL ลมพิษขึ้นหนาและตัว BP 75/40 mmHg, RR 40/min, pulse 110/mon, no wheezing, GI C. Serum C3 33 mg/dL D. Urine Na 40 mmol/L increased bowel sounds E. Urine protein to creatinine 5 mg/mg A. Direct release mediator from mast cell 101. เด็กหญิงอายุ 10 ป มีอาการปสสาวะบอย และปสสาวะเปนสีแดง รวมกับมีไข ไอมา B. Complement deactivation 2-3 วัน กอนหนานี้เคยเปน 1 ครั้งหายแลว ตรวจรางกายปกติ Urinalysis พบ pH 6.0, WBC C. Disturbance of arachidonic acid metabolism 5-10/HP, RBCสูง รวมกับ dysmorphic RBC จงใหการวินิจฉัย D. Type I hypersensitivity A. Hypercalcuri B. Renal calculi E. Eosinophil degranulation Comprehensive - Pedriatrics หนาที่ 26 Comprehensive - Pedriatrics หนาที่ 27
  • 16. 105. เด็กหญิง asthma มา 5 ป มาดวยอาการหอบเหนื่อย P110/min, BP 140/70mmHg, <<<<< Cardiology >>>>> rhonchi ทั้ง 2 ปอด ได O2 + sulbutamol NB ควรใหอะไรอีก 108. เด็กหญิงอายุ 12 ป มาโรงพยาบาลดวยอาการใจสั่น ตรวจรางกายพบ T 37oC, P A. IV Dexa B. IV terbutaline 200/min, BP 100/70 mmHg ให EKG มาดังรูป จงใหการรักษา C. Theophylline C. NB ipratropium E. NB beclomethasone 106. เด็กชายอายุ 8 ป มี VSD ใหเลือดกอนผาตัด ขณะไดเลือดไป 5 ml ผูปวยมีอาการหอบ เหนื่อย wheezing, BP ลดจาก 90/50 เปน 60/20 ไมมีไข หลังจากหยุดใหเลือดแลว ทานจะ ทําอะไรตอ A. Adrenaline B. Antihistamine C. Dexamethasone D. Respiratory support E. ตรวจสอบเลือดที่ให A. Adenosine 0.1 mg/kg IV B. Digitalis 0.1 mg/kg IV C. Verapamil 0.01 mg/kg IV D. DC synchronized cardioversion <<<<< Psychiatry >>>>> E. CPR 107. เด็กหญิงอายุ 7 ป มาดวยปวดทอง คลื่นไส ปสสาวะราดตอนกลางคืน กลางวันปกติ มี 109. เด็กอายุ 3 ป ไข อาเจียน 4วัน ตับโต มีจุดเลือดออก BP 90/70 mmHg, check จากอะไร อารมณหงุดหงิดฉุนเฉียว แขนขาออนแรง ตรวจรางกายพบวา T 38.5oC, BP 100/70 A. Decreased H/S concentration B. Decreased sympathetic activity mmHg, GA: inactive, resp.: dyspnea with fruity odor, Ix: hyponatremia, K4.5 จง C. Decreased cardiac contraction D. Decreased preload investigation ที่เหมาะสม E. Increased afterload A. blood glucose B. blood lactose 110. เด็กอายุ 7 ป มีไข เหนื่อย หอบ ปวดขอ, มีไข PR&RR ปกติ, ไมมีผล lung, CVS พบ C. U/S whole abdomen D. CT brain systolic ejection murmur 2/6 ที่ apex กับ diastolic blowing murmur 2/6 ที่ apex บอกวา E. UA นาจะมีภาวะใด A. AR B. PR Comprehensive - Pedriatrics หนาที่ 28 Comprehensive - Pedriatrics หนาที่ 29
  • 17. C. MR D. MR with relative MS <<<<< Hematology >>>>> E. AR กับ AS 114. เด็กอายุ 8 ขวบ มีอาการไขสูง, H/C พบ Gm neg rod, มี bleeding ไมหยุดที่ central line, เจาะเลือดพบ PT, aPTT, TT prolong ทั้งหมด ทานคิดวาจะเจอผล lab เปนอยางไร A. Fibrinogen ต่ํา D-dimer สูง B. Fibrinogen ต่ํา D-dimer ต่ํา <<<<< RS >>>>> C. Fibrinogen สูง D-dimer สูง D. fibrinogen สูง D-dimer ต่ํา 111. สงสัย croup ใหการรักษา.... 115. เด็กอายุ 6 เดือน ชัก ซึม มีไข กระหมอมโปง, T 39°C, prolonged PT & aPTT, normal <<<<< GI >>>>> TT, เด็กคนนี้มีความผิดปกติที่ Factor ใด 112. ผูปวยเด็กมี URI symptoms, 2 วันตอมามีอาการทองเสีย อุจจาระเปนน้ํามากกวาเนื้อ A. VII B. IX รองไหงอแง ตรวจรางกายพบ moderate dehydration ขอใดถูก C. XI D. Prothrombin complex A. ตรวจลมหายใจพบ H2, CH4 116. ชายอายุ 7 ป Hx birth wt 1500g, เหนื่อย เพลีย ซีด, CBC พบ RBC normal with B. ในลําไสจะพบ CO2, NH3 เพิ่มขึ้น polychromasia นิดหนอย มามโต ตับไมโต DX C. Stool exam พบ lactic acid เพิ่มขึ้น A. Congenital spherocytosis D. Stool exam ไมพบ reducing substance B. Portal vein thrombosis E. ใหดื่ม ORS อยางเดียวจนกวาจะหาย diarrhea 113. เด็กชายอายุ 4 เดือน มีไขและอุจจาระเหลวมา 2 วัน อาเจียน อุจจาระเปนน้ําพุง เปนลม <<<<< Endocrine >>>>> มาก ซึมจากการขาดน้ํา คิดวาติดเชื้ออะไร 117. ผูปวยเด็กหญิงอายุ 14 ป มีอาการใจสั่น คอโต น้ําหนักลด, T3 & T4 สูง TSH ต่ํา, A. Shigella B. Salmonella I131uptake สูง, thyroid scan: diffused enlargement, จงใหการวินิจฉัย C. Cholera D. Rotavirus A. Follicular CA B. … E. Norovirus C. Lymphoma D. Grave’s disease 118. เด็กหญิงอายุหนึ่งขวบครึ่ง แมพามารับวัคซีน แพทยตรวจรางกายอื่นๆ ปกติ ยกเวน genitalia พบ - Rt.Labioscrotal fold มี mild rugae และ คลําได mass ขนาดประมาณ 1 cm - Lt Labioscrotal fold ปกติ คลําไดmass ขนาดประมาณ 1 cm ที่ low inguinal canal Comprehensive - Pedriatrics หนาที่ 30 Comprehensive - Pedriatrics หนาที่ 31
  • 18. - Phallus 1.5 cm length & มี urethral opening ที่ perineum <<<<< Neurology >>>>> จงวินิจฉัย 122. เด็กอายุ 8 เดือนมีไขมา 3 สัปดาห 1 สัปดาหกอนน้ําหนักลด 1 kg เด็กรองงอแง A. Hypospadias with undescended testis drowsy anterior frontanel บวม Meningeal irritation +ve PE : T 38.5 Bw 9 Kg B. Male pseudohermaphrodite LP : Cell 300 (L 95%), Sugar 20(Blood 80), Protein 200 C. Female pseudohermaphrodite จงใหการวินิจฉัย D. CAH A. Bacterial meningitis B. Viral meningitis 119. มารดาพา หญิง 13 ป มาดวย ปจด.ไมสม่ําเสมอ เริ่มมี ปจด.มาตั้งแตอายุ 12 ป 3 เดือน C. Fungal meningitis D. Eosinophilic meningitis มาเพียง 2 ครั้ง ครั้งละ 5-7 วัน กะปริดกะปรอย ไมมีปวดทองนอย ครั้งลาสุด 2 สัปดาห E. TB meningitis กอน PE: breast and pubic hair tanner stage 3, other unremarkable ทานจะใหคําแนะนํา 123. เด็กอายุ 15 เดือน ยาพามาปรึกษาเรื่องพัฒนาการชา เด็กพูดคุยได เลนไดดี นั่งเองได หรือรักษาอยางไร เกาะยืนไดบาง สังเกตวาเด็กไมคอยใชแขนขวา เวลายกตัวขาขวาจะเหยียดเกร็ง ตรวจ A. ตรวจเลือด B. สงหาสูตินรีแพทย หาสาเหตุ C. แนะนําวาเปนภาวะปกติที่พบได D. ใหยาพวกฮอรโมนปรับ ปจด. รางกาย เด็กเลนไดดี รูสึกตัวดี มี Lt hemiparesis, increased muscle tone, hyperreflexia 120. เด็กชายอายุ 10 ป มาพบแพทยดวยเรื่องตัวเตี้ย โตชา แข็งแรงดี พอสูง 175 cm แมสูง ของแขนซาย (โจทยมันใหขางซายขวามาอยางงี้เลยงงๆ) นึกถึงโรคใดมากที่สุด 152 cm เด็กพัฒนาการปกติ ไมมีโรค ขณะนี้ BW at 10th percentile, Ht 125 cm (P3) A. Brain tumour B. Child neglect 1 ปที่ผานมาโตขึ้น 5 cm, x-ray bone age = 8 yr C. Cerebral palsy D. Hydrocephalus A. Familial short stature B. Constitutional delayed growth E. Increased intracranial pressure C. Malnutrition D. Cushing’s syndrome 124. เด็ก 4 เดือน ซึม ไข ชัก มากอน สรุป meningitis อะไรสําคัญสุด E. GH deficiency A. LP B. CBC 121. เด็กอายุ 7 ป มีน้ําหนักขึ้น 7 kg ภายใน 5 เดือน สวนสูงเพิ่มขึ้น(จําตัวเลขไมได) ตรวจ รางกาย มี acne (signs of androgen เพิ่มขึ้น) C. CT brain A. Late onset CAH B. Adrenal tumor 125. ชายอายุ 10 ป ขา 2 ขางออนแรงและชา 3 วันเดินไมได อุจจาระปสสาวะไดปกติ C. ACTH releasing tumor motor power: lower limbs grade 0, upper limbs grade 2, lesion อยูที่ใด A. Brainstem B. Peripheral nerve Comprehensive - Pedriatrics หนาที่ 32 Comprehensive - Pedriatrics หนาที่ 33
  • 19. C. Muscle D. Spinal cord A. Erythema infectiosum B. Roseolar infantum E. Cerebral hemisphere 129. เด็กหญิงอายุ 17 วัน แมพามา รพ. เพราะมีผื่นมา 2 วัน PE: multiple discreted tiny 126. หญิง 12 ป มาพบแพทยดวยอาการขาออนแรงทั้งสองขางมา 1 วัน โดยเริ่มมีอาการ erythematous papules on forehead and neck, ตรวจ G/S + Wright’s stain ไมพบ bacteria ออนแรงตั้งแตเมื่อวาน ตื่นเชามาพบวาขาไมมีแรง ไมสามารถกลั้นปสสาวะอุจจาระได PE or WBC คิดวาเปนไรนิ พบ hypotonia both lower extremities, motor power 0/V, areflexia, upper extremities A. Milia B. Miliaria rubra normal tone, lose of sensation below umbilicus, loose anal sphincter tone, Dx? C. Bullous pemphigoid D. Erythema toxicum neonatorum A. Transverse myelitis B. Poliomyelitis E. Seborrheic dermatitis C. Guillan-Barre syndrome D. Cerebrovascular accident 130. เด็กอายุ 7 ป เปนตุมสีเดียวกับเนื้อ ผิวขรุขระ ขึ้นกระจายตามแขน ขนาด 0.3-0.5 cm เปนมา 6 เดือน สังเกตวาพอเกาแลวตุมจะกระจายไปตามรอยเกา Dx? <<<<< Genetics >>>>> A. Melanocytic nevus B. Molluscum contagiosum 127. เด็กแรกเกิด ใหลักษณะมาเปน Down’s syndrome สุดๆ แมอายุ 27 ป พออายุ 27 ป ขอ C. Seborrehic keratosis D. Chicken pox ใดถูก E. Verrucus valgaris A. ไมตองตรวจ chromosome พอแม ถาตรวจลูกไดเปน trisomy B. นาจะเกิดจาก translocation มากกวา trisomy C. ถาเกิดจาก unbalanced translocation จะมีโอกาสในลูกคนตอไป 50% D. ไมตองตรวจ chromosome เนื่องจากแมอายุนอยกวา 35 ป <<<<< Dermatology >>>>> 128. เด็กอายุ 10 ป มีไขมา 3 วัน มีผื่นเปน confluent erythematous rash at both cheek มี reticulated …. ? Give diagnosis Comprehensive - Pedriatrics หนาที่ 34 Comprehensive - Pedriatrics หนาที่ 35
  • 20. - - - - - - - - - - - 03 Obstetrics & Gynecology - - - - - - - - - - - 135. หญิงตั้งครรภ น้ําเดินมา 2 hr. กอนมาโรงพยาบาล ตรวจพบน้ําเดินจริง PV: station 0, dilate 4 cm eff 90% หลังจากนั้น 2 ชั่วโมง station 1+, dilate 5 cm effacement 100% <<<<< Obstetrics >>>>> มี earlydece จง management 131. มาคลอด เฝาการคลอด PV ครั้งแรก effacement 90%, os 5 cm. อีก 1 ชั่วโมง A. Drip oxytocin B. C/S because arrest of dilatation effacement 100%, os 7 cm ทําไง C. C/S station +1 D. Amnioinfusn ไมมี choice มาใหอะ 136. หญิง 20ป G1P0 GA38wk มาดวยปวดครรภสม่ําเสมอ 3-5นาที นานครั้งละ 60วิ ตรวจ 132. หญิงอายุ 33 ป ตั้งครรภ GA 33 wk. เด็กในทองไมไดดิ้นมา 6 wk u/s ไมเจอheart beat พบ dilate 3 cm, efface 90% แพทยทําการเจาะถุงน้ําคร่ําเรงคลอด และใหepidural infusion ตรวจพบมี Hct 34%, plt 70,000, PT17, PTT44, TT22 จะใหอะไร คุมอาการปวด 1ชม. ตอมา PV: dilate 5cm eff100% ทําอยางไรตอ A. cryoprecipitate B. FFP A. ใหoxytocinเรงคลอด เพราะprotract active phase C. Traxanemic acid D. low dose heparin B. observeตอ การคลอดเปนไปตามปกติ E. Plt. concentration C. หยุด epidural infusion ใหปากมดลูกขยายไดดีขึ้น 133. หญิงมาฝากครรภคิดวาตั้งครรภ 26 wks มากฝากครรภครั้งแรก F/U พบวาครรภมี D. เริ่มเบงได ขนาดเล็กกวา แพทยตรวจ GA นอยไป 5wk ขอใดไมใชสาเหตุของ IUGR E. C/S A. จําปจดผิด B. Thalassemia 137. หญิง ตั้งครรภ 24 สัปดาห มาดวยพบแพทยดวยอาการซึม ซีด สับสน ตรวจ Lab Hb 6, C. แมติดเชื้อ D. GDM class B1 WBC 12000 (Neutrophil 35%, Lymphocyte 65%) Platelet 15000, Coomb test- negative, E. chromosome abnormality BUN 60, Cr 3.5, LDH 3000 Peripheral Blood smear- Schistocyte 2+ 134. หญิงตั้งครรภแรก GA 32 wk เมื่อ 2 hr กอน มีเลือดออกทางชองคลอดปริมาณมาก จงใหการวินิจฉัย ผาอนามัย 7-8 ผืนชุม มีทองแข็งเปนพักๆ มาตรวจพบ mildly pale VS stable FHR 140 มี A. Preecclampsia contraction ทุก 5 นาที ตรวจ PV มีเลือดคางอยู 20 ml ในชองคลอด เลือดหยุดไหล แลวเคย B. Acute Disseminated intravascular coagulopathy มีประวัติเลือดออกทางชองคลอดเมื่อ 1 เดือนกอนและ 2 wk กอน C. HELLP syndrome A. tocolytic drugs B. oxytocin augmentation D. ITP C. PV ประเมิน BISHOP score D. Abdominal ultrasound E. TTP 138. หญิงเพิ่งคลอดลูก day3 ตอนนี้มีfluctuant nodule ที่breast ซาย ถามmx Comprehensive - Obstetrics & Gynecology หนาที่ 36 Comprehensive - Obstetrics & Gynecology หนาที่ 37